The Praxis Series™ eBooks

The Official

Study Guide Teaching Reading Test Test Code: 0204 Revised 2011

Study Topics



Practice Questions Directly from the Test Makers



Test-Taking Strategies



www.ets.org/praxis This ebook was issued to Chelissa Gaines, order #10789211026. Unlawful distribution of this ebook is prohibited.

Study Guide for the Teaching Reading Test ▲























A PUBLICATION OF ETS

This ebook was issued to Chelissa Gaines, order #10789211026. Unlawful distribution of this ebook is prohibited.

Copyright © 2011 by Educational Testing Service. All rights reserved. ETS, the ETS logo, GRE, and LISTENING. LEARNING. LEADING. are registered trademarks of Educational Testing Service (ETS) in the United States and other countries. PRAXIS and THE PRAXIS SERIES are trademarks of ETS. SAT is a registered trademark of the College Entrance Examination Board.

This ebook was issued to Chelissa Gaines, order #10789211026. Unlawful distribution of this ebook is prohibited.

Table of Contents Study Guide for the Teaching Reading Test

























This ebook was issued to Chelissa Gaines, order #10789211026. Unlawful distribution of this ebook is prohibited.

TA B L E O F C O N T E N T S

Chapter 1 Introduction to the Test and Suggestions for Using This Study Guide . . . . . . . . . . . . . . . . . . . . . . . . 1

Chapter 2 Background Information on The Praxis Series™ Assessments . . . . . . . . . . . . . . . . . . . . . . . . . . . . 5

Chapter 3 Don’t Be Defeated by Multiple-Choice Questions . . . . . . . . . . . . . . . . . . . . . . . . . . . . . . . . . . . . . . . 8

Chapter 4 Succeeding on the Constructed-Response Questions . . . . . . . . . . . . . . . . . . . . . . . . . . . . . . . . . . . . 14

Chapter 5 Preparing for the Constructed-Response Part of the Test . . . . . . . . . . . . . . . . . . . . . . . . . . . . . . . . . 20

Chapter 6 Study Topics . . . . . . . . . . . . . . . . . . . . . . . . . . . . . . . . . . . . . . . . . . . . . . . . . . . . . . . . . . . . . . . . . . . . 26

Chapter 7 Practice Questions . . . . . . . . . . . . . . . . . . . . . . . . . . . . . . . . . . . . . . . . . . . . . . . . . . . . . . . . . . . . . . . 33

Chapter 8 Right Answers and Explanations . . . . . . . . . . . . . . . . . . . . . . . . . . . . . . . . . . . . . . . . . . . . . . . . . . . . 53

Chapter 9 Sample Responses and How They Were Scored . . . . . . . . . . . . . . . . . . . . . . . . . . . . . . . . . . . . . . . . 67

Chapter 10 Are You Ready? Last-Minute Tips . . . . . . . . . . . . . . . . . . . . . . . . . . . . . . . . . . . . . . . . . . . . . . . . . . . 74

Appendix A Study Plan Sheet . . . . . . . . . . . . . . . . . . . . . . . . . . . . . . . . . . . . . . . . . . . . . . . . . . . . . . . . . . . . . . . . 77

Appendix B For More Information . . . . . . . . . . . . . . . . . . . . . . . . . . . . . . . . . . . . . . . . . . . . . . . . . . . . . . . . . . . . 79

This ebook was issued to Chelissa Gaines, order #10789211026. Unlawful distribution of this ebook is prohibited.

Chapter 1

Introduction to the Test and Suggestions for Using This Study Guide

























This ebook was issued to Chelissa Gaines, order #10789211026. Unlawful distribution of this ebook is prohibited.

CHAPTER 1

Introduction to the Teaching Reading Test The Praxis™ Teaching Reading test is designed to support both • Licensing an entry-level reading teacher, and • Adding a reading endorsement to an existing license. Teaching Reading is designed for individuals whose preparatory program has included intensive training in the teaching of reading. The test content reflects the five essential components of effective reading instruction as identified by the National Reading Panel: Phonemic Awareness, Phonics, Fluency, Comprehension, and Vocabulary. Test questions call on the individual’s knowledge of reading theory and practice as well as the ability to apply knowledge and principles to instructional situations. The test taker will be required to analyze and respond to situations involving both classes and individual students for students at grade levels from kindergarten through high school, including students with diverse needs. Note: The Teaching Reading test does not assume that the test taker has graduate-level preparation to be a reading specialist. It does not test the consulting, coordinating, and supervisory roles that might be part of the responsibility of a licensed reading specialist. The Teaching Reading test (0204) consists of 90 multiple-choice questions and 3 constructed response items. These cover 6 major areas, in the following proportions:

Approximate Number of Questions

Approximate Percentage of Examination

• Emergent Literacy

15

12%

• Phonological Awareness

14

12%

• Alphabetic Principle/Phonics and Word Analysis

14

12%

• Comprehension and Fluency

30

25%

• Vocabulary

17

14%

3 constructed response

25%

Content Category

• Instructional Processes

Test takers have 2 hours to complete the test. The test is not intended to assess teaching skills but rather fundamental knowledge in the major areas of Reading.

2

Study Guide for the Teaching Reading Test

This ebook was issued to Chelissa Gaines, order #10789211026. Unlawful distribution of this ebook is prohibited.

CHAPTER 1

Suggestions for using the “Study Topics” chapter of this study guide This test is different from a final exam or other tests you may have taken for other courses because it is comprehensive—that is, it covers material you may have learned in several courses during your entire undergraduate program. It requires you to synthesize information you have learned from many sources and to understand the subject as a whole. Therefore, you should review and prepare for it, rather than merely becoming familiar with the question formats. A thorough review of the material covered on the test will significantly increase your likelihood of success. Moreover, studying for your licensing exam is a great opportunity to reflect on and develop a deeper understanding of reading content and pedagogical knowledge and methods before you begin to teach or to reflect on previous teaching experience. As you prepare to take the test, it may be particularly helpful for you to think about how you would apply the study topics and sample exercises to the practical experience you obtained during your teacher preparation program. Your student teaching experience will be especially relevant to your thinking about the materials in the study guide. We recommend the following approach for using the “Study Topics” chapters to prepare for the test.

Become familiar with the test content. Learn what will be assessed in the test, covered in chapters 5 and 6.



Assess how well you know the content in each area. After you learn what topics the test contains, you should assess your knowledge in each area. How well do you know the material? In which areas do you need to learn more before you take the test? It is quite likely that you will need to brush up on most or all of the areas.



Develop a study plan. Assess what you need to study and create a realistic plan for studying. You can develop your study plan in any way that works best for you. A “Study Plan” form is included in appendix A at the end of the book as a possible way to structure your planning. Remember that this is a licensure test and covers a great deal of material. Plan to review carefully. You will need to allow time to find the books and other materials, time to read the material and take notes, and time to go over your notes.



Identify study materials. Most of the material covered by the test is contained in standard introductory textbooks. If you do not own introductory texts that cover all the areas, you may want to borrow some from friends or from a library. You may also want to obtain a copy of your state’s standards for Reading. (One way to find these standards quickly is to go to the website for your state’s Department of Education.) The textbooks used in secondary classrooms may also prove useful to you, since they also present the material you need to know. Use standard school and college introductory textbooks and other reliable, professionally prepared materials. Don’t rely heavily on information provided by friends or from searching the Internet. Neither of these sources is as uniformly reliable as textbooks.



Work through your study plan. You may want to work alone, or you may find it more helpful to work with a group or with a mentor. Work through the topics and questions provided in chapters 5 and 6. Rather than memorizing definitions from books, be able to define and discuss the topics in your own words and understand the relationships between diverse topics and concepts. If you are working with a group or mentor, you can also try informal quizzes and questioning techniques.



Proceed to the practice questions. Once you have completed your review, you are ready to benefit from the “Practice Test” portion of this guide.













Study Guide for the Teaching Reading Test

This ebook was issued to Chelissa Gaines, order #10789211026. Unlawful distribution of this ebook is prohibited.

3

CHAPTER 1

Suggestions for using the “Practice Test,” “Right Answers and Explanations,” and “Sample Responses and How They Were Scored” chapters

Read chapter 3. This chapter will sharpen your skills in reading and answering multiple-choice questions. For you to succeed on multiple-choice questions, you must focus carefully on the question, avoid reading things into the question, pay attention to details, and sift patiently through the answer choices.



Read chapter 4. This chapter will sharpen your skills in reading and answering constructed-response questions.



Answer the practice questions in chapter 7. Work on the practice questions in a quiet place without distractions. Remember that the practice questions are only examples of the way the topics are covered in the test. The test will have different questions.



Score the practice questions. Go through the detailed answers in chapter 8 and the sample responses in chapter 9 and mark the questions you answered correctly and the ones you missed. Look over the explanations of the questions you missed and see if you understand them.



Decide whether you need more review. After you have looked at your results, decide whether there are areas that you need to brush up on before taking the actual test. Go back to your textbooks and reference materials to see if the topics are covered there. You might also want to go over your questions with a friend or teacher who is familiar with the subjects.



Assess your readiness. Do you feel confident about your level of understanding in each of the areas? If not, where do you need more work? If you feel ready, complete the checklist in chapter 10 (“Are You Ready?”) to double-check that you’ve thought through the details. If you need more information about registration or the testing situation itself, use the resources in appendix B: “For More Information.”













Note: Every effort is made to provide the most recent information in this study guide. However, The Praxis Series™ tests are continually evaluated and updated. You will always find the most recent information about this test, including the topics covered, number of questions, time allotted, and scoring criteria, in the Test at a Glance booklet available online at http://www.ets.org/praxis/prepare/materials.

4

Study Guide for the Teaching Reading Test

This ebook was issued to Chelissa Gaines, order #10789211026. Unlawful distribution of this ebook is prohibited.

Chapter 2

Background Information on The Praxis Series™ Assessments

























This ebook was issued to Chelissa Gaines, order #10789211026. Unlawful distribution of this ebook is prohibited.

CHAPTER 2

What Are The Praxis Series™ Subject Assessments? The Praxis Series™ Subject assessments are designed by Educational Testing Service (ETS) to assess your knowledge of best practices in the teaching of reading. They are a part of the licensing procedure in many states. This study guide covers an assessment that tests your knowledge of the actual content you will be expected to teach once you are licensed. Your state has adopted The Praxis Series tests because it wants to confirm that you have achieved a specified level of mastery in your subject area before it grants you a license to teach in a classroom. The Praxis Series tests are part of a national testing program, meaning that the test covered in this study guide is required in more than one state for licensure. The advantage of a national program is that if you want to move to another state, you can transfer your scores from one state to another. However, each state has specific test requirements and passing scores. If you are applying for a license in another state, you will want to verify the appropriate test and passing score requirements. This information is available online at www.ets.org/praxis/states or by calling ETS at 800-772-9476 or 609-771-7395.

What Is Licensure? Licensure in any area—medicine, law, architecture, accounting, cosmetology—is an assurance to the public that the person holding the license possesses sufficient knowledge and skills to perform important occupational activities safely and effectively. In the case of teacher licensing, a license tells the public that the individual has met pre-defined competency standards for beginning teaching practice. Because a license makes such a serious claim about its holder, licensure tests are usually quite demanding. In some fields, licensure tests have more than one part and last for more than one day. Candidates for licensure in all fields plan intensive study as part of their professional preparation: some join study groups, others study alone. But preparing to take a licensure test is, in all cases, a professional activity. Because it assesses the entire body of knowledge for the field you are entering, preparing for a licensure exam takes planning, discipline, and sustained effort.

Why Does My State Require The Praxis Series Assessments? Your state chose The Praxis Series assessments because the tests assess the breadth and depth of content— called the “domain”—that your state wants its teachers to possess before they begin to teach. The level of content knowledge, reflected in the passing score, is based on recommendations of panels of teachers and teacher educators in each subject area. The state licensing agency and, in some states, the state legislature ratify the passing scores that have been recommended by panels of teachers.

6

Study Guide for the Teaching Reading Test

This ebook was issued to Chelissa Gaines, order #10789211026. Unlawful distribution of this ebook is prohibited.

CHAPTER 2

What Do the Tests Measure? The Praxis Series Subject assessments are tests of content knowledge. They measure your understanding and skills in a particular subject area. Multiple-choice tests measure a broad range of knowledge across your content area. Constructed-response tests measure your ability to provide in-depth explanations of a few essential topics in a given subject area. Content-specific pedagogy tests, most of which are constructedresponse, measure your understanding of how to teach certain fundamental concepts in a subject area. The tests do not measure your actual teaching ability, however. They measure your knowledge of a subject and of how to teach it. The teachers in your field who help us design and write these tests, and the states that require them, do so in the belief that knowledge of your subject area is the first requirement for licensing. Teaching combines many complex skills, only some of which can be measured by a single test. While the test covered in this study guide measures content knowledge, your teaching ability is a skill that is typically measured in other ways; for example, through observation, videotaped practice, or portfolios.

How Were These Tests Developed? ETS began the development of The Praxis Series Subject assessments with a survey. For each subject, teachers around the country in various teaching situations were asked to judge which knowledge and skills a beginning teacher in that subject needs to possess. Professors in schools of education who prepare teachers were asked the same questions. These responses were ranked in order of importance and sent out to hundreds of teachers for review. All of the responses to these surveys (called “job analysis surveys”) were analyzed to summarize the judgments of these professionals. From their consensus, we developed guidelines, or specifications, for the multiple-choice and constructed-response tests. Each subject area had a committee of practicing teachers and teacher educators who wrote the specifications, which were reviewed and eventually approved by teachers. From the test specifications, groups of teachers and professional test developers created test questions that met content requirements and satisfied the ETS Standards for Quality and Fairness.** When your state adopted The Praxis Series Subject assessments, local panels of practicing teachers and teacher educators in each subject area met to examine the tests and to evaluate each question for its relevance to beginning teachers in your state. This is called a “validity study” because local practicing teachers validate that the test content is relevant to the job. For the test to be adopted in your state, teachers in your state must judge that it is valid. During the validity study, the panel also provides a passing-score recommendation. This process includes a rigorous review to determine how many of the test questions a beginning teacher in that state would be able to answer correctly. Your state’s licensing agency then reviewed the panel’s recommendations and made a final determination of the passing-score requirement. Throughout the development process, practitioners in the teaching field—teachers and teacher educators— participated in defining what The Praxis Series Subject assessments would cover, which test would be used for licensure in your subject area, and what score would be needed to achieve licensure. This practice is consistent with how professional licensure works in most fields: those who are already licensed oversee the licensing of new practitioners. When you pass The Praxis Series Subject assessments, you and the practitioners in your state will have evidence that you have the knowledge and skills required for beginning teaching practice.

** ETS Standards for Quality and Fairness (2003, Princeton, NJ) are consistent with the “Standards for Educational and Psychological Testing,” industry standards issued jointly by the American Educational Research Association, the American Psychological Association, and the National Council on Measurement in Education (1999, Washington, DC).

Study Guide for the Teaching Reading Test

This ebook was issued to Chelissa Gaines, order #10789211026. Unlawful distribution of this ebook is prohibited.

7

Chapter 3

Don’t Be Defeated by Multiple-Choice Questions

























This ebook was issued to Chelissa Gaines, order #10789211026. Unlawful distribution of this ebook is prohibited.

CHAPTER 3

Understanding Multiple-Choice Questions When you read multiple-choice questions on the Praxis Teaching Reading test, you will probably notice that the syntax (word order) is different from the word order you’re used to seeing in ordinary material that you read, such as newspapers or textbooks. One of the reasons for this difference is that many test questions contain the phrase “which of the following.” To answer a multiple-choice question successfully, you need to consider carefully the context set up by the question and limit your choice of answers to the list given. The purpose of the phrase “which of the following” is to remind you to do this. For example, look at this question.

Which of the following is a flavor made from beans? (A) Strawberry (B) Cherry (C) Vanilla (D) Mint

You may know that chocolate and coffee are also flavors made from beans, but they are not listed, and the question asks you to select from the list that follows (“which of the following”). So the answer has to be the only bean-derived flavor in the list: vanilla. Notice that the answer can be substituted for the phrase “which of the following.” In the question above, you could insert “vanilla” for “which of the following” and have the sentence “Vanilla is a flavor made from beans.” Sometimes it helps to cross out “which of the following” and insert the various choices. You may want to give this technique a try as you answer various multiple-choice questions on the practice test. Looking carefully at the “which of the following” phrase helps you to focus on what the question is asking you to find and on the answer choices. In the simple example above, all of the answer choices are flavors. Your job is to decide which of the flavors is the one made from beans. The vanilla bean question is pretty straightforward. But the phrase “which of the following” can also be found in more challenging questions. Look at this question:

Which of the following teacher actions would best support the development of literacy skills in the early elementary classroom? (A) Using a commercially developed language arts curriculum aligned with national standards (B) Creating attractive bulletin boards using commercially prepared materials (C) Teaching phonics, decoding, and word-recognition skills using work sheets (D) Providing children with motivating reading and writing materials and assignments

The placement of “which of the following” tells you that the list of choices is a list of examples (in this case, these are examples of things a teacher might do in an early elementary classroom). What are you supposed to find as an answer? You are supposed to find the choice that best supports the development of literacy skills.

Study Guide for the Teaching Reading Test

This ebook was issued to Chelissa Gaines, order #10789211026. Unlawful distribution of this ebook is prohibited.

9

CHAPTER 3

Educational Testing Service (ETS) question-writers and editors work very hard to word each question as clearly as possible. Sometimes, though, it helps to put the question in your own words. Here, you could paraphrase the question as “Which of these techniques would help my students most with their literacy skills?” The correct answer is (D). (Research has found that providing students with varied opportunities to engage in the reading and writing processes stimulates their motivation and advances their reading skills. Teaching phonics skills in isolation using work sheets may teach letter-sound relationships, but not the literacy skills of reading and writing. There is little indication that commercial curriculums and commercially produced classroom materials help students learn to read.) You may also find that it helps you to circle or underline each of the critical details of the question in your test book so that you don’t miss any of them. It’s only by looking at all parts of the question carefully that you will have all of the information you need to answer it. Circle or underline the critical parts of what is being asked in this question.

Which of the following strategies will best foster multiculturalism in the classroom? (A) Playing a variety of music from different cultures (B) Providing foods from various countries in the role-play area (C) Inviting families to share their customs with the class (D) Hanging travel posters from various countries in the classroom

Here is one possible way you may have annotated the question:

Which of the following strategies will best  foster multiculturalism  in the classroom? (A) Playing a variety of music from different cultures (B) Providing foods from various countries in the role-play area (C) Inviting families to share their customs with the class (D) Hanging travel posters from various countries in the classroom

After thinking about the question, you can probably see that you are being asked to look at a list of classroom techniques and decide which one gives students the greatest awareness of other cultures. The correct answer is (C). The important thing is understanding what the question is asking. With enough practice, you should be able to determine what any question is asking. Knowing the answer is, of course, a different matter, but you have to understand a question before you can answer it correctly.

10

Study Guide for the Teaching Reading Test

This ebook was issued to Chelissa Gaines, order #10789211026. Unlawful distribution of this ebook is prohibited.

CHAPTER 3

Be Familiar with Multiple-Choice Question Types You will probably see more than one question format on a multiple-choice test. Here are examples of some of the more common question formats.

1.

Complete the statement

In this type of question, you are given an incomplete statement. You must select the choice that will make the completed statement correct.

After using the words “tooth” and “teeth” correctly, some children begin saying “tooths” and “teeths.” This usage results from (A) paralanguage (B) difficulty in producing language (C) overgeneralization (D) poor listening skills

To check your answer, reread the question and add your answer choice at the end. Be sure that your choice best completes the sentence. The correct answer is (C).

2.

Which of the following

This question type is discussed in detail in a previous section. The question contains the details that must be satisfied for a correct answer, and it uses “which of the following” to limit the choices to the four choices shown, as this example demonstrates.

Jimmy, who is in preschool, is able to take off and put on his coat independently during the school day. However, when his mother picks him up, he expects her to help him get dressed. Which of the following would be the most appropriate teacher comment in this situation? (A) “May I ask why are you putting Jimmy’s coat on for him?” (B) “If you put on Jimmy’s coat, he will not do it for himself.” (C) “Jimmy can now put on his coat by himself.” (D) “It’s important that you let Jimmy take care of himself.”

The correct answer is (C).

3.

Other formats

New formats are developed from time to time to find new ways of assessing knowledge with multiple-choice questions. If you see a format you are not familiar with, read the directions carefully. Then read and approach the question the way you would any other question, asking yourself what you are supposed to be looking for and what details are given in the question that help you find the answer.

Study Guide for the Teaching Reading Test

11

This ebook was issued to Chelissa Gaines, order #10789211026. Unlawful distribution of this ebook is prohibited.

CHAPTER 3

Other Useful Facts About the Test 1. You can answer the questions in any order. You can go through the questions from beginning to end, as many test takers do, or you can create your own path. Perhaps you will want to answer questions in your strongest area of knowledge first and then move from your strengths to your weaker areas. There is no right or wrong way. Use the approach that works best for you. 2. There are no trick questions on the test. You don’t have to find any hidden meanings or worry about trick wording. All of the questions on the test ask about subject matter knowledge in a straightforward manner. 3. Don’t worry about answer patterns. There is one myth that says that answers on multiple-choice tests follow patterns. There is another myth that there will never be more than two questions with the same lettered answer following each other. There is no truth to either of these myths. Select the answer you think is correct based on your knowledge of the subject. 4. There is no penalty for guessing. Your test score for multiple-choice questions is based on the number of correct answers you have. When you don’t know the answer to a question, try to eliminate any obviously wrong answers and then guess at the correct one. 5. It’s OK to write in your test booklet. You can work out problems right on the pages of the booklet, make notes to yourself, mark questions you want to review later, or write anything at all. Your test booklet will be destroyed after you are finished with it, so use it in any way that is helpful to you. But make sure to mark your answers on the answer sheet.

12

Study Guide for the Teaching Reading Test

This ebook was issued to Chelissa Gaines, order #10789211026. Unlawful distribution of this ebook is prohibited.

CHAPTER 3

Smart Tips for Taking the Test 1. Put your answers in the right “bubbles.” It seems obvious, but be sure that you are filling in the answer “bubble” that corresponds to the question you are answering. A significant number of test takers fill in a bubble without checking to see that the number matches the question they are answering. 2. Skip the questions you find extremely difficult. There are sure to be some questions that you think are hard. Rather than trying to answer these on your first pass through the test, leave them blank and mark them in your test booklet so that you can come back to them later. Pay attention to the time as you answer the rest of the questions on the test, and try to finish with 10 or 15 minutes remaining so that you can go back over the questions you left blank. Even if you don’t know the answer the second time you read the questions, see if you can narrow down the possible answers, and then guess. 3. Keep track of the time. Bring a watch to the test, just in case the clock in the test room is difficult for you to see. You will probably have plenty of time to answer all of the questions, but if you find yourself becoming bogged down in one section, you might decide to move on and come back to that section later. 4. Read all of the possible answers before selecting one—and then reread the question to be sure the answer you have selected really answers the question being asked. 5. Check your answers. If you have extra time left over at the end of the test, look over each question and make sure that you have filled in the “bubble” on the answer sheet as you intended. Many test takers make careless mistakes that they could have corrected if they had checked their answers. 6. Don’t worry about your score when you are taking the test. No one is expected to answer all of the questions correctly. Your score on this test is not analogous to your score on the SAT®, the GRE®, or other similar-looking (but in fact very different!) tests. It doesn’t matter on this test whether you score very high or barely pass. If you meet the minimum passing scores for your state and you meet the state’s other requirements for obtaining a teaching license, you will receive a license. In other words, your actual score doesn’t matter, as long as it is above the minimum required score. With your score report you will receive a booklet entitled Understanding Your Praxis Scores, which lists the passing scores for your state.

Study Guide for the Teaching Reading Test

13

This ebook was issued to Chelissa Gaines, order #10789211026. Unlawful distribution of this ebook is prohibited.

Chapter 4

Succeeding on the Constructed-Response Questions

























This ebook was issued to Chelissa Gaines, order #10789211026. Unlawful distribution of this ebook is prohibited.

CHAPTER 4

This chapter provides advice for maximizing your success on the Teaching Reading test, with special focus on the scoring guides and procedures used by the scorers. Chapters 5 and 6 offer step-by-step strategies for working through questions, lists of the topics covered, and lists of sources you can use to prepare.



Advice from the experts Scorers who have scored hundreds of real tests were asked to give advice to teacher candidates planning to take the Teaching Reading constructed-response test. The scorers’ advice boiled down to the practical pieces of advice described below. 1. Read and answer the question accurately. Be sure to dissect the parts of the question and analyze what each part is asking you to do. If the question asks you to describe or discuss, keep those requirements in mind when composing your response—do not just give a list. 2. Answer everything that is asked in the question. This seems simple, but many test takers fail to provide a complete response. If a question asks you to do three distinct things in your response, don’t give a response to just two of those things. No matter how well you write about those two things, the scorers will not award you full credit. 3. Give a thorough and detailed response. Your response must indicate to the scorers that you have a thorough understanding of the applicable principles and guidelines related to teaching reading. The scorers will not read into your response any information that is not specifically stated. If something is not written, they do not know that you know it and will not give you credit for it.

A word of caution: Superfluous writing will obscure your points and will make it difficult for the scorers to be confident of your full understanding of the material. Be straightforward in your response. Do not try to impress the scorers. If you do not know the answer, you cannot receive full credit, but if you do know the answer, provide enough information to convince the scorers that you have a full understanding of the topic.

4. Do not change the question or challenge the basis of the question. Stay focused on the question that is asked. You will receive no credit or, at best, a low score if you choose to answer another question or if you state, for example, that there is no possible answer. Answer the question by addressing the fundamental issues. Do not venture off-topic to demonstrate your particular field of expertise if it is not specifically related to the question. This undermines the impression that you understand the concept adequately. 5. Reread your response, both to improve your writing (for the Constructed-response test) and to check that you have written what you thought you wrote. Frequently, sentences are left unfinished or clarifying information is omitted.

Study Guide for the Teaching Reading Test

15

This ebook was issued to Chelissa Gaines, order #10789211026. Unlawful distribution of this ebook is prohibited.

CHAPTER 4

The General Scoring Guide for the Teaching Reading Test The scorers’ advice on the preceding page corresponds with the official scoring criteria used at scoring sessions. It is a good idea to be familiar with the scoring rubric so that you can maximize your success and spend your time on things that matter (e.g., demonstrating understanding of the selection and providing good examples) rather than spending time on things that don’t matter (e.g., writing a very long essay, making copious citations). The following scoring rubric provides the overarching framework for scoring the questions in the constructed response portion of the Teaching Reading test. Each question on the test is scored on a scale from 0 to 3. The response is considered in its entirety when the scorer assigns the score. The same general scoring guide is used to score Questions 91–93.

GENERAL SCORING GUIDE This scoring guide is used to evaluate responses to reading and reading instruction questions. The score range is 0 to 3.

Score 3 The response demonstrates a thorough understanding of the elements of reading and reading instruction relevant to the question. A response in this category •

Clearly and specifically answers all parts of the question, addressing all crucial aspects of the instructional situation described



Shows strong knowledge of concepts, theories, facts, procedures or methodologies relevant to the question. Any errors of fact or terminology are minor and do not detract from the understanding shown.



Provides a strong explanation that is well supported by relevant evidence.

Score 2 The response demonstrates a basic or general understanding of the elements of reading and reading instruction relevant to the question. A response in this category

16



Adequately answers most or all parts of the question in a way that is appropriate for the instructional situation described



Shows basic or general knowledge of concepts, theories, facts, procedures or methodologies relevant to the question. Any errors do not detract from the general understanding shown.



Provides a basic explanation that is adequately supported by relevant evidence.

Study Guide for the Teaching Reading Test

This ebook was issued to Chelissa Gaines, order #10789211026. Unlawful distribution of this ebook is prohibited.

CHAPTER 4

Score 1 The response demonstrates a weak or limited understanding of the elements of reading and reading instruction relevant to the question. A response in this category •

Answers some parts of the question at a basic level



Demonstrates one or more of the following weaknesses $$ Fails to answer most parts of the question and/or fails to address crucial aspects of the instructional situation described $$ Shows weak or limited knowledge of concepts, theories, facts, procedures or methodologies relevant to the question. The weakness may be indicated by errors or misconceptions. $$ Any explanation provided is weak and inadequately supported by evidence.

Score 0 The response demonstrates minimal (or no) understanding of the elements of reading and reading instruction relevant to the question. A response in this category •

Fails to respond appropriately to any part of the question.



Shows no knowledge of concepts, theories, facts, procedures or methodologies relevant to the question; or, any information presented about reading and reading instruction is seriously mistaken.

Also receiving a score of 0 would be responses that are blank, completely off topic, or not written in English

What You Should Know About How the Constructed Response Portion of Teaching Reading Test Is Scored As you build your skills in writing answers to constructed-response questions, it is important to have in mind the process used to score the tests. If you understand the process by which experts determine your scores, you may have a better context in which to think about your strategies for success.

How the Tests are Scored After each test administration, test books are returned to ETS. The test booklets in which constructedresponse answers are written are sent to the location of the scoring session. The scoring sessions usually take place over two days. The sessions are led by scoring leaders, highly qualified reading teachers who have many years of experience scoring test questions. All of the remaining scorers are experienced reading teachers and reading teacher-educators. An effort is made to balance experienced scorers with newer scorers at each session; the experienced scorers provide continuity with past sessions, and the new scorers ensure that new ideas and perspectives are considered and that the pool of scorers remains large enough to cover the test’s needs throughout the year.

Study Guide for the Teaching Reading Test

17

This ebook was issued to Chelissa Gaines, order #10789211026. Unlawful distribution of this ebook is prohibited.

CHAPTER 4

Preparing to Train the Scorers The scoring leaders meet several days before the scoring session to assemble the materials for the training portions of the main session. Training scorers is a rigorous process, and it is designed to ensure that each response gets a score that is consistent both with the scores given to other papers and with the overall scoring philosophy and criteria established for the test when it was designed. The scoring leaders first review the “General Scoring Guides,” which contain the overall criteria, stated in general terms, for awarding the appropriate score. The leaders also review and discuss—and make additions to, if necessary—the “Question-Specific Scoring Guides,” which serve as applications of the general guide to each specific question on the test. The question-specific guides cannot cover every possible response the scorers will see, but they are designed to give enough examples to guide the scorers in making accurate judgments about the variety of answers they will encounter. To begin identifying appropriate training materials for an individual question, the scoring leaders first read through many responses to get a sense of the range of answers. They then choose a set of benchmarks, one paper at each score level. These benchmarks serve as solid representative examples of the kind of response that meets the scoring criteria at each score level and are considered the foundation for score standards throughout the session. The scoring leaders then choose a larger set of test-taker responses to serve as sample papers. These sample papers represent the wide variety of possible responses that the scorers might see. The sample papers serve as the basis for practice scoring at the scoring session, so that the scorers can rehearse how they will apply the scoring criteria before they begin. The process of choosing a set of benchmark responses and a set of sample responses is followed systematically for each question to be scored at the session. After the scoring leaders are done with their selections and discussions, the sets they have chosen are photocopied and inserted into the scorers’ folders in preparation for the session.

Training at the Main Scoring Session At the scoring session, the scorers are placed into groups according to the question they are assigned to score. New scorers are distributed equally across all groups. One of the scoring leaders is placed with each group. The “Chief Scorer” is the person who has overall authority over the scoring session and plays a variety of key roles in training and in ensuring consistent and fair scores. For each question, the training session proceeds in the same way: 1. All scorers carefully read through the question they will be scoring. 2. All scorers review the “General Scoring Guide” and the “Question-Specific Scoring Guide” for the question. 3. For each question, the leader guides the scorers through the set of benchmark responses, explaining in detail why each response received the score it did. Scorers are encouraged to ask questions and share their perspectives.

18

Study Guide for the Teaching Reading Test

This ebook was issued to Chelissa Gaines, order #10789211026. Unlawful distribution of this ebook is prohibited.

CHAPTER 4

4. Scorers then practice on the set of sample responses chosen by the leader. The leader polls the scorers on what scores they would award and then leads a discussion to ensure that there is a consensus about the scoring criteria and how they are to be applied. 5. When the leader is confident that the scorers for that question will apply the criteria consistently and accurately, the actual scoring begins.

Quality-Control Processes A number of procedures are followed to ensure that accuracy of scoring is maintained during the scoring session. Most importantly, each response is scored twice, with the first scorer’s decision hidden from the second scorer. If the two scores for a paper are the same or differ by only one point, the scoring for that paper is considered complete, and the test taker will be awarded the sum of the two scores. If the two scores differ by more than one point, the response is scored by a scoring leader, who has not seen the decisions made by the other two scorers. If this third score is midway between the first two scores, the test taker’s score for the question is the sum of the first two scores; otherwise, it is the sum of the third score and whichever of the first two scores is closer to it. Another way of maintaining scoring accuracy is through back-reading. Throughout the session, the leader for each question checks random samples of scores awarded by all the scorers. If the leader finds that a scorer is not applying the scoring criteria appropriately, that scorer is given more training. At the beginning of the second day of reading, additional sets of papers are scored using the consensus method described above. This helps ensure that the scorers are refreshed on the scoring criteria and are applying them consistently. Finally, the scoring session is designed so that several different scorers (usually 6) contribute to any single test taker’s total score. This minimizes the effects of a scorer who might score slightly more stringently or generously than other scorers. The entire scoring process—general and specific scoring guides, standardized benchmarks and samples, consensus scoring, adjudication procedures, back-reading, and rotation of test questions to a variety of scorers—is applied consistently and systematically at every scoring session to ensure comparable scores for each administration and across all administrations of the test. Given the information above about how constructed responses are scored and what the scorers are looking for in successful responses, you are now ready to look at specific questions, suggestions of how to approach the questions, and sample responses and scores given to those responses.

Study Guide for the Teaching Reading Test

19

This ebook was issued to Chelissa Gaines, order #10789211026. Unlawful distribution of this ebook is prohibited.

Chapter 5

Preparing for the Constructed-Response Part of the Test ▲























This ebook was issued to Chelissa Gaines, order #10789211026. Unlawful distribution of this ebook is prohibited.

CHAPTER 5

The goal of this chapter is to provide you with strategies for how to read, analyze, and understand the questions on the Constructed Response part of the Teaching Reading test and then how to outline and write successful responses.

Introduction to the Question Types Part B of the Teaching Reading test consists of three constructed-response questions that require the candidate to demonstrate the ability to apply knowledge and theory of reading to specific students’ needs and classroom scenarios. Each task will be scored on a 0–3 scale.

What to Study Success on this test is not simply a matter of learning more about how to respond to constructed-response questions. It also takes real knowledge of the field. As mentioned above, the test is designed to gather evidence about your knowledge of reading theory and practice as well as the ability to apply knowledge and principles to instructional situations. It therefore would serve you well to review your notes and texts from your teaching of reading courses. The following websites are particularly relevant to the types of knowledge topics and skills covered by the test. These websites also provide lists of books and access to journal articles on reading that may be helpful. http://www.nationalreadingpanel.org/ http://www.reading.org/General/Default.aspx http://www.ncte.org/

Understanding What the Questions Are Asking It is impossible to write a successful response to a question unless you thoroughly understand the question. Often test takers jump into their written response without taking enough time to analyze exactly what the question is asking, how many different parts of the question need to be addressed, and how the information in the accompanying tables needs to be addressed. The time you invest in making sure you understand what the question is asking will very likely pay off in a better performance, as long as you budget your time and do not spend a large proportion of the available time just reading the question. Examine the overall question closely, then identify what specific questions are being asked, mentally organize your response, and outline your key themes. Leave yourself plenty of time to write your answer. If you think out your response beforehand, your essay will probably be stronger.

Study Guide for the Teaching Reading Test

21

This ebook was issued to Chelissa Gaines, order #10789211026. Unlawful distribution of this ebook is prohibited.

CHAPTER 5

Sample Question To illustrate the importance of understanding the question before you begin writing, let’s start with a sample question: Directions: Read the scenario and then respond to all parts of the task on the lined pages provided. The suggested time to spend on this question is 10 minutes. Scenario: The early childhood teachers in a school have decided to implement interdisciplinary science units with a focus on reading. The district has implemented a policy that all schools must include a technology component that will support reading instruction. Task: Be sure to respond to both of the following: • •

Briefly describe an instructional material the teachers would use within these units to support the reading focus. Explain the criteria used to evaluate the material in meeting these instructional needs. Briefly describe an activity using technology with the interdisciplinary science units. Specify how the activity supports student reading goals.

Organizing Your Response Successful responses start with successful planning, either with an outline or with another form of notes. By planning your response, you greatly decrease the chances that you will forget to answer any part of the question. You increase the chances of creating a well-organized response, which is something the scorers look for. Your note-taking space also gives you a place to jot down thoughts whenever you think of them— for example, when you have an idea about one part of the question while you are writing your response to another part. Like taking time to make sure you understand what the question is asking, planning your response is time well invested, although you must keep track of the time so that you leave sufficient time to write your response. To illustrate a possible strategy for planning a response, let us focus again on the sample question introduced in the previous section. We analyzed the question and found that it asked for a two-part response. You might begin by jotting down those parts on your notes page, leaving space under each. This will ensure that you address each part when you begin writing.

22

Study Guide for the Teaching Reading Test

This ebook was issued to Chelissa Gaines, order #10789211026. Unlawful distribution of this ebook is prohibited.

CHAPTER 5

Sample Notes—Main Parts to be Answered Here you start by identifying each part of the question: •

Reading material related to science unit °



How to evaluate the selected material’s ability to successfully support reading focus

Integrating technology to support reading focus

You then might quickly fill out the main ideas you want to address in each part, like this:

Sample Notes—Ideas under Each Main Part •

Reading material related to science unit °

Tradebooks ▪



How to evaluate the selected material’s ability to successfully support reading focus •

Different readability levels to support all students at their current reading level



Connect to the science topic being studied



The tradebooks present accurate and current information

Integrating technology to support reading focus °

WebQuests or online labs—support reading skill development (i.e., cause and effect), content-related vocabulary development, and support comprehension of what was read in the tradebooks

To earn the highest number of points from the scorers, you will need to do all of the following: ▪

Answer all parts of the question.



Give reasons for your answers.



Demonstrate subject-specific knowledge in your answer.



Refer to the data in the stimulus.

Now look at your notes and add any ideas that would address these characteristics. You have now created the skeleton of your written response.

Study Guide for the Teaching Reading Test

23

This ebook was issued to Chelissa Gaines, order #10789211026. Unlawful distribution of this ebook is prohibited.

CHAPTER 5

Writing Your Response Now the important step of writing your response begins. The scorers will not consider your notes when they score your paper, so it is crucial that you integrate all the important ideas from your notes into your actual written response. Some test takers believe that every written response on a Praxis test has to be in formal essay form—that is, with an introductory paragraph, then paragraphs with the response to the question, then a concluding paragraph. This is the case for very few Praxis tests (e.g., English and Writing). The Teaching Reading test does not require formal essays, so you should use techniques that allow you to communicate information efficiently and clearly. For example, you can use bulleted or numbered lists, or a chart, or a combination of essay and chart.

Sample Response that Received a Score of 3 Teachers can use tradebooks to help integrate reading into a science unit. The instructional material that is needed is a variety of books on the differing reading levels of the students. An example of this would be teaching a science unit on deserts. The teacher would collect books at different readability levels on living in a desert, animals in a desert, or any other related topics to use with the class. The criteria used to ensure the books meet instructional needs are that they are at an appropriate reading level for the students, have a connection to the topic being studied, and contain accurate information. A technological activity the students could do would be an online net lab. The students would, for example, be required to follow written instructions as they completed a simulation of a lab activity such as the dissection of an owl pellet. Reading skills such as cause and effect could be reinforced with a webquest through which students discovered and observed relationships between actions. Related hands-on activities could be linked to the webquest tasks. By using these methods of integrating science and reading, the teachers can meet the technology component mandated by the district and further support students’ reading development.

Commentary on Sample Response that Earned a Score of 3 This response answers each part of each task fully and completely, providing relevant supporting details.

Sample Response that Received a Score of 2 I would begin by finding books on insects. Every day we would read books and discuss reading techniques and skills. We would also collect bugs and caterpillars to study. Using PowerPoint, we would make slides that compared and contrasted different insects as a class. Each student would then choose a bug to study and make a slide with a picture and sentence about their insect. I will evaluate the students’ ability to use PowerPoint and make notes on which students need a little more help. We will then make a CD with all the insect slides and burn copies of it so that each student can take one home. We will then have a short quiz on the reading to check for comprehension.

Commentary on Sample Response that Earned a Score of 2 Although most parts of the tasks are addressed, the response did not include how the instructional material was evaluated.

24

Study Guide for the Teaching Reading Test

This ebook was issued to Chelissa Gaines, order #10789211026. Unlawful distribution of this ebook is prohibited.

CHAPTER 5

Sample Response that Received a Score of 1 An instructional material that teachers would use for the unit could be how a butterfly grows. The kids are learning reading and science at the same time. They could have a computer in the classroom and have Leapfrog Interactive Reading on the computer. The technology would be helping the kids to read and they would be having fun while doing it.

Commentary on Sample Response that Earned a Score of 1 This response shows a limited knowledge of choosing appropriate instructional materials. No supporting evidence or explanations are provided. Not all parts of the task are addressed.

In Conclusion Whatever format you select, the important thing is that your answer be thorough, complete, and detailed. You need to be certain that you do the following: ▪

Answer all parts of the question.



Give reasons for your answers.



Demonstrate subject-specific knowledge in your answer.



Refer to the data in the stimulus.

It is a good idea to use the practice test in the next chapter to help you develop a plan for how you will take the test on the actual testing day, especially if you tend to get nervous or tend to freeze up in a testing situation. Whatever format you select for your response, the important thing is that your answer be thorough, complete, and detailed.

Study Guide for the Teaching Reading Test

25

This ebook was issued to Chelissa Gaines, order #10789211026. Unlawful distribution of this ebook is prohibited.

Chapter 6 Study Topics

























This ebook was issued to Chelissa Gaines, order #10789211026. Unlawful distribution of this ebook is prohibited.

CHAPTER 6

Introduction to the Test The Teaching Reading test is designed to support licensing entry-level reading teachers and adding a reading endorsement to an existing license. Teaching Reading is designed for individuals whose preparatory program has included intensive training in the teaching of reading. The test content reflects the five essential components of effective reading instruction as identified by the National Reading Panel: Phonemic Awareness, Phonics, Fluency, Comprehension, and Vocabulary. Test questions call on the individual’s knowledge of reading theory and practice as well as the ability to apply knowledge and principles to instructional situations. The test taker will be required to analyze and respond to situations involving both classes and individual students at grade levels kindergarten through high school, including students with diverse needs. This chapter is intended to help you organize your preparation for the test and to give you a clear indication about the depth and breadth of the knowledge required for success on the test.

Study Guide for the Teaching Reading Test

27

This ebook was issued to Chelissa Gaines, order #10789211026. Unlawful distribution of this ebook is prohibited.

CHAPTER 6

Using the topic lists that follow

Special questions

You are not expected to be an expert on all aspects of the topics that follow. You should understand the major characteristics of each topic, recognize the minor topics, and have some familiarity with the subtopics. Virtually all accredited undergraduate Reading Education programs address the majority of these topics, subtopics, and even minor topics.

Interspersed throughout the list of topics are questions that are intended to help you test your knowledge of fundamental concepts and your ability to apply fundamental concepts to situations in the classroom or the real world. Most of the questions require you to combine several pieces of knowledge to formulate an integrated understanding and response. If you spend time on these questions, you will gain increased understanding and facility with the subject matter covered on the test. You might want to discuss these questions and your answers with a teacher or mentor.

Here, for instance, is one of the topic lists in “Concepts of Print” under “Emergent Literacy”: ▶ Concepts of Print • What is the relationship between speech and print? • How do environmental print, pictures, and symbols contribute to literacy development? • What are some strategies for teaching letter recognition? • How can students’ print awareness be assessed? Referring to textbooks, state standards documents, or other sources as needed, make sure you can describe the topics in your own words. You are likely to find that the topics below are covered by most introductory reading education textbooks and textbooks for related fields, but a general survey textbook may not cover all of the subtopics. Consult materials and resources, including lecture and laboratory notes, from all your reading coursework. You should be able to match up specific topics and subtopics with what you have covered in your courses. Try not to be overwhelmed by the volume and scope of content knowledge in this guide. An overview such as this that lists reading topics does not offer you a great deal of context. Although a specific term may not seem familiar as you see it here, you might find you could understand it when applied to a reallife situation. Many of the items on the actual test will provide you with a context to apply to these topics or terms, as you will see when you look at the practice questions in Chapter 7.

28

Note that the questions are not short-answer or multiple-choice, and this study guide does not provide the answers. The questions are intended as study questions, not practice questions. Thinking about the answers to them should improve your understanding of fundamental concepts and will probably help you answer a broad range of questions on the test. For example, the following question appears in the list of study topics under “Comprehension and Fluency”: 1. What is the relationship between students’ background knowledge and comprehension? If you think about this question, perhaps jotting down some notes about background knowledge and comprehension, you will have probably prepared yourself to answer multiple-choice questions similar to the one below: Sample Question: A class is preparing to read expository text about spiders. The students brainstorm ideas about spiders and make a visual representation of the concepts and relationships. The primary purpose of this activity is to (A) have the students predict what the text will be about (B) activate and develop students’ prior knowledge about the topic (C) give the students a purpose for reading the material (D) help the students decide on what they are going to read aloud

Study Guide for the Teaching Reading Test

This ebook was issued to Chelissa Gaines, order #10789211026. Unlawful distribution of this ebook is prohibited.

CHAPTER 6

The correct answer is (B). Graphic organizers are visual displays that enable children to organize and represent meanings, ideas, and concepts on a topic. They can be used for many purposes, including activating prior knowledge on a topic to be studied or read about. Pappas et al, An Integrated Language Perspective in the Elementary School, 1990, p.273. Since these students are preparing to read, the purpose of the graphic organizer is to activate prior knowledge.

Study questions about this topic may include: 1. What are ways that teachers can model the rules of Standard English while respecting regional and dialectical variations?

Study Point B. Concepts of Print ⦁⦁

Interrelatedness between print and speech ◾◾

Topic 1. Emergent Literacy

–– Language experience approach –– Shared writing

Study Point A. Oral Language ⦁⦁

⦁⦁

Interrelatedness between oral language and reading skills such as phonemic awareness, phonics, vocabulary, fluency, and comprehension. Expressive and receptive components ◾◾

◾◾

⦁⦁

◾◾

⦁⦁

–– Interactive writing ⦁⦁

⦁⦁

◾◾

Phonological

◾◾

Syntactic

◾◾

Semantic

◾◾

Pragmatic

Appropriate techniques to assess students’ oral language development

Study Guide for the Teaching Reading Test

Importance of modeling concepts of print

◾◾

⦁⦁

Language variations in the classroom

Knowledge of four language systems

Contribution of environmental print, pictures, and symbols to literacy development

◾◾

Vocabulary

–– Dialectical differences, English as a Second Language speakers, non-Standard English ⦁⦁

–– Morning message

Expectations at each stage of oral language development

Environmental influences that affect oral language development

Strategies to increase students’ awareness that print is speech written down

⦁⦁

One-to-one correspondence Directionality of print (left to right/top to bottom/front to back)

Environmental influences that affect students’ development of print awareness Importance of students’ ability to differentiate between ◾◾

Words and spaces

◾◾

First and last letters

◾◾

Punctuation marks

◾◾

Words and sentences

Study questions about this topic may include: 1. What are some instructional strategies for teaching letter recognition? 2. How can students’ print awareness be assessed?

29

This ebook was issued to Chelissa Gaines, order #10789211026. Unlawful distribution of this ebook is prohibited.

CHAPTER 6

Topic 2. Phonological Awareness ⦁⦁

Relationship between phonemic awareness and the development of decoding and encoding skills ◾◾

⦁⦁

⦁⦁

⦁⦁

⦁⦁

Sound matching and sound isolation

◾◾

Auditory blending and segmenting of sounds

◾◾

Invented spelling

Onsets and rimes

◾◾

Decoding words when reading

Systematic and explicit instructional strategies for teaching phonological awareness skills

◾◾

Digraphs

Diphthongs: both vowels form one syllable –– “a” and “i” in “rail” or “oy” in toy

◾◾

⦁⦁

Schwa sound

Difference between phonetically regular and irregular words ◾◾

Strategies for teaching both types –– Word sorts

⦁⦁

1. At which age ranges should the various phonological skills be acquired? 2. What does the term “phonemic awareness” mean?

⦁⦁

3. How is phonemic awareness related to phonological awareness?

⦁⦁

4. What is the relationship between phonemic awareness and the development of decoding and encoding skills?

Developmental stages that readers progress through when learning to decode and encode (spell)

Blends

–– Vowel Digraphs: ea (sea), ee (feet), oa (boat), oo (moon) or (book), aw (claw), ow (cow) or (glow), ew (new), ou (cloud)

Study questions about this topic may include:

⦁⦁

◾◾

–– Consonant Digraphs: th, sh, wh, and ch

Assessment of students’ phonological awareness

Topic 3. Alphabetics/Phonics and Word Analysis

Letter-sound correspondences

◾◾

Progression of phonological awareness skills ◾◾

◾◾

–– bl, dr, str, pl, sm, gr

Ability to orally identify and later produce a matching rhyming word

◾◾

Progression of phonics instruction

⦁⦁

Knowledge of syllables ◾◾

Syllable types

◾◾

Syllabication principles

Systematic and explicit strategies for teaching phonics and word analysis Strategies for reading and spelling multisyllabic words using meaningful units ◾◾

Morphemes

◾◾

Syllables

◾◾

Accenting principles

Assessment of students’ phonics and word analysis skills 1. What is the difference between phonics and phonological awareness? 2. What role do decodable text, writing practice, and spelling practice play in reinforcing specific phonics skills?

30

Study Guide for the Teaching Reading Test

This ebook was issued to Chelissa Gaines, order #10789211026. Unlawful distribution of this ebook is prohibited.

CHAPTER 6

Topic 4. Comprehension and Fluency

Study Point B. Fluency ⦁⦁

Study Point A. Comprehension ⦁⦁

Relationship between background knowledge and comprehension ◾◾

Components of fluency ◾◾

⦁⦁

Assessment of students’ prior knowledge

Strategies to develop and improve fluency ◾◾

–– KWL chart –– Semantic map ◾◾

◾◾

Strategies to activate students’ prior knowledge Strategies to build students’ background knowledge for unfamiliar subject matter

⦁⦁

⦁⦁

⦁⦁

⦁⦁

Repeated use of word lists, phrases, and passages at appropriate instructional levels

◾◾

Modeling fluent reading

◾◾

Paired reading

◾◾

Echo reading

◾◾

Readers Theater

Role vocabulary plays in comprehension

Informal and formal methods of assessing reading fluency

Systematic, explicit instruction to develop comprehension skills

1. What instructional strategies can be used to improve oral reading fluency?

⦁⦁ ⦁⦁

Accuracy, appropriate pace, automaticity, and prosody

◾◾

Directed Reading-Thinking Activity (DR-TA)

◾◾

Graphic organizers

Flexible use of multiple reading strategies to enhance reading comprehension ◾◾

When and why to apply a particular strategy

◾◾

Self-monitoring strategies

2. What is the relationship between fluency and comprehension?

Topic 5. Vocabulary ⦁⦁

Direct and indirect methods of vocabulary instruction ◾◾

Effect of language structure on comprehension Role of text features (headings, subheadings) and genre in reading comprehension

◾◾

Semantic maps –– Need for materials with rich contextual support for vocabulary development

1. How can writing activities be used to support reading comprehension?

◾◾

Word analysis –– Root words, prefixes, suffixes

2. In what ways do students’ cultural and linguistic backgrounds affect comprehension?

–– Etymology ⦁⦁

3. What factors should be taken into account when choosing texts for students?

Importance of providing a wide range of reading opportunities

Word relationships ◾◾

Antonyms –– Opposite meanings: hot and cold

Study Guide for the Teaching Reading Test

31

This ebook was issued to Chelissa Gaines, order #10789211026. Unlawful distribution of this ebook is prohibited.

CHAPTER 6 ◾◾

◾◾

Synonyms

◾◾

–– Similar meanings: smart and intelligent

–– Educational software

Homonyms

–– Word processing

–– Sound or look the same, but have different meanings

–– Digital storytelling –– Multimedia

a. bear and bare b. wind and wind ⦁⦁

⦁⦁

The effect of grammatical functions and word forms on meaning Methods for assessing and monitoring vocabulary development

–– Audio books –– Educational blogs ⦁⦁

Assessment ◾◾

–– Informal assessments-running record, miscue analysis, Informal Reading Inventory (IRI), anecdotal notes, observation, portfolio, interview, conference, rubric, checklist, writing sample

2. How can teachers model the use of context as a strategy to confirm word meaning? 3. How do common prefixes, suffixes, and roots affect the meaning of English words?

Topic 6. Instructional Processes Instructional Practices ◾◾

Creation of a learning environment that supports literacy development –– Incorporation of motivational strategies that promote active student engagement

◾◾

Strategies for differentiating instruction –– Grouping strategies a. flexible grouping

⦁⦁

Curriculum Material ◾◾ ◾◾

Types of assessments –– Formal assessments-standardized tests

1. What criteria should be used when selecting words for vocabulary instruction?

⦁⦁

Integration of technology to support literacy

–– Individual and group administered assessments ◾◾

Purposes and proper uses of the various assessment types –– Determine independent level, instructional level, and frustration level –– Types of miscues being made –– Strengths and weaknesses –– Impact on instruction –– Developmentally appropriate

1. How can teachers use assessment data to inform their instruction? 2. What are effective ways to communicate students’ reading progress to stakeholders (i.e., parents and administration)?

Various uses for different types of texts Selection of instructional materials that reflect societal diversity –– Multicultural texts –– Recognition of stereotypes

32

Study Guide for the Teaching Reading Test

This ebook was issued to Chelissa Gaines, order #10789211026. Unlawful distribution of this ebook is prohibited.

Chapter 7 Practice Questions

























This ebook was issued to Chelissa Gaines, order #10789211026. Unlawful distribution of this ebook is prohibited.

CHAPTER 7

Now that you have studied the content topics and have worked through strategies relating to multiple-choice and constructed-response questions, you should take the following practice test. You will probably find it helpful to simulate actual testing conditions, giving yourself 70 minutes to work on the questions. You can print out and use the answer sheet provided if you wish. Keep in mind that the test you take at an actual administration will have different questions, although the proportion of questions in each area and major subarea will be approximately the same. You should not expect the percentage of questions you answer correctly in these practice questions to be exactly the same as when you take the test at an actual administration, since numerous factors affect a person’s performance in any given testing situation. When you have finished the practice questions, you can score your answers and read the explanations of the best answer choices in chapter 8.

34

Study Guide for the Teaching Reading Test

This ebook was issued to Chelissa Gaines, order #10789211026. Unlawful distribution of this ebook is prohibited.

TEST NAME:

Teaching Reading Practice Questions

Time—70 Minutes 66 Multiple-Choice and 2 constructed-response questions (Note, at the official test administration, there will be 90 multiple-choice and 3 constructedresponse questions, and you will be allowed 120 minutes to complete the test.)

This ebook was issued to Chelissa Gaines, order #10789211026. Unlawful distribution of this ebook is prohibited.

Q

O

E

F

D

E

D

E

B

This ebook was issued to Chelissa Gaines, order #10789211026. Unlawful distribution of this ebook is prohibited.

R

S

T

U

V

W

X

Y

Z

S

T

U

V

W

X

Y

Z

Z

Y

X

W

V

U

T

S

R

Q

P

O

N

M

J

I

H

Z

Y

X

W

V

U

T

S

R

Q

P

O

N

M

L

K

J

I

H

G

F

E

D

C

B

A

Z

Y

X

W

V

U

T

S

R

Q

P

O

N

M

L

K

J

I

H

G

F

E

D

C

B

A

Z

Y

X

W

V

U

T

S

R

Q

P

O

N

M

L

K

J

I

H

G

F

E

D

C

B

A

Z

Y

X

W

V

U

T

S

R

Q

P

O

N

M

L

K

J

I

H

G

F

E

D

C

B

A

O

Q

2

3

4

5

6

7

8

9

2

3

4

5

6

7

8

9

0

1

0

1

9

8

7

6

5

4

3

2

1

0

9

8

7

6

5

4

3

2

1

0

9

8

7

6

5

4

3

2

1

0

9

8

7

6

5

4

3

2

1

0

9

8

7

6

5

4

3

2

1

0

5. CANDIDATE ID NUMBER

SIGNATURE:

TELEPHONE NUMBER:

(Print)

MAILING ADDRESS:

0

9

8

7

6

5

4

3

2

1

(

) Home

TEST DATE:

(

First Name (Given)

City

Center Number

Country

Center Name

)

Apt. # (if any)

M. I.

Q3338-01

Business

0 1

1

9

8

7

6

5

4

3

2

1

0

Zip or Postal Code

State or Province

State or Province

Room Number

6. TEST CENTER/REPORTING LOCATION

Country

City

P.O. Box or Street Address

Last Name (Family or Surname)

Copyright © 2009 by Educational Testing Service. All rights reserved. ETS and the ETS logo are registered trademarks of Educational Testing Service (ETS) in the United States and other countries. Praxis and The Praxis Series are trademarks of ETS.

Q

R

P

P

Q

O

O

N

N

L

L

K

K

M

J

J

L

I

I

M

K

H

H

G

F

G

F

G

D

C

B

C

B

A

A

C

A

FI

(Print)

YOUR NAME:

Last Name (first 6 letters)

2.

Enter your last name and first initial. Omit spaces, hyphens, apostrophes, etc.

Jan. Feb. Mar. April May June July Aug. Sept. Oct. Nov. Dec.

Month

9

8

7

6

5

4

3

2

1

0

2

9

8

7

6

5

4

3

2

1

0

9

8

7

6

5

4

3

2

1

0

3

9

8

7

6

5

4

3

2

1

0

9

8

7

6

5

4

3

2

1

0

0

3

3

4

9

8

7

6

5

4

3

2

1

0

5

2

2

2 1

1

6 7

0 1

9

0

9

8

7

6

5

4

3

2

1

0

9

8

7

6

5

4

3

2

1

0

9

8

7

6

5

4

3

2

1

0

9

8

7

6

5

4

3

2

1

0

9

8

7

6

5

4

3

2

1

0

9

8

7

6

5

4

3

2

1

0

9

8

7

6

5

4

3

2

1

0

9

8

7

6

5

4

3

2

1

752107

80394-62263 • TF99E12.250 • Printed in U.S.A.

9. TEST FORM

S

8. TEST BOOK SERIAL NUMBER

8 9

5

8

6 7

4

4

3

0

0

1

PAGE 1

4. SOCIAL SECURITY NUMBER

S123456789T

Day

3. DATE OF BIRTH

7. TEST CODE/ FORM CODE

Use only a pencil with soft black lead (No. 2 or HB) to complete this answer sheet. Be sure to fill in completely the circle that corresponds to the proper letter or number. Completely erase any errors or stray marks.

1. NAME

DO NOT USE INK

Answer Sheet Y

DO NOT WRITE IN THIS AREA.

1

2

3

4

5

6

7

8

9 10 11 12 13 14 15 16 17 18 19 20 21 22 23 24 25 26 27 28 29 30 31 32 33 34 35 36 37 38 39 40 41 42 43 44 45 46 47

PAGE 2

CERTIFICATION STATEMENT: (Please write the following statement below. DO NOT PRINT.) “I hereby agree to the conditions set forth in the Registration Bulletin and certify that I am the person whose name and address appear on this answer sheet.”

SIGNATURE:

DATE: Month

1. 2. 3. 4. 5. 6. 7. 8. 9. 10. 11. 12. 13. 14. 15. 16. 17. 18. 19. 20. 21. 22. 23. 24. 25. 26. 27. 28. 29. 30.

A

B

C

D

A

B

C

D

A

B

C

D

A

B

C

D

A

B

C

D

A

B

C

D

A

B

C

D

A

B

C

D

A

B

C

D

A

B

C

D

A

B

C

D

A

B

C

D

A

B

C

D

A

B

C

D

A

B

C

D

A

B

C

D

A

B

C

D

A

B

C

D

A

B

C

D

A

B

C

D

A

B

C

D

A

B

C

D

A

B

C

D

A

B

C

D

A

B

C

D

A

B

C

D

A

B

C

D

A

B

C

D

A

B

C

D

A

B

C

D

ETS USE ONLY

31. 32. 33. 34. 35. 36. 37. 38. 39. 40. 41. 42. 43. 44. 45. 46. 47. 48. 49. 50. 51. 52. 53. 54. 55. 56. 57. 58. 59. 60.

1R

A

B

C

D

A

B

C

D

A

B

C

D

A

B

C

D

A

B

C

D

A

B

C

D

A

B

C

D

A

B

C

D

A

B

C

D

A

B

C

D

A

B

C

D

A

B

C

D

A

B

C

D

A

B

C

D

A

B

C

D

A

B

C

D

A

B

C

D

A

B

C

D

A

B

C

D

A

B

C

D

A

B

C

D

A

B

C

D

A

B

C

D

A

B

C

D

A

B

C

D

A

B

C

D

A

B

C

D

A

B

C

D

A

B

C

D

A

B

C

D

2R

61. 62. 63. 64. 65. 66. 67. 68. 69. 70. 71. 72. 73. 74. 75. 76. 77. 78. 79. 80. 81. 82. 83. 84. 85. 86. 87. 88. 89. 90.

3R

TR

A

B

C

D

A

B

C

D

A

B

C

D

A

B

C

D

A

B

C

D

A

B

C

D

A

B

C

D

A

B

C

D

A

B

C

D

A

B

C

D

A

B

C

D

A

B

C

D

A

B

C

D

A

B

C

D

A

B

C

D

A

B

C

D

A

B

C

D

A

B

C

D

A

B

C

D

A

B

C

D

A

B

C

D

A

B

C

D

A

B

C

D

A

B

C

D

A

B

C

D

A

B

C

D

A

B

C

D

A

B

C

D

A

B

C

D

A

B

C

D

ESSAY

Day

Year

91. Write your response on the appropriate pages of the answer sheet.

92. Write your response on the appropriate pages of the answer sheet.

93. Write your response on the appropriate pages of the answer sheet.

TUESDAY

CS

This ebook was issued to Chelissa Gaines, order #10789211026. Unlawful distribution of this ebook is prohibited.

Q3338-2

Cor

CHAPTER 7

TEACHING READING Time allowed for the whole test (Part A and Part B)—120 minutes Part A 90 Multiple-Choice Questions (Suggested time—60 minutes) Directions: Each of the questions or incomplete statements below is followed by four choices (A, B, C, and D). Choose the best response from the choices for each question and fill in the appropriate space for that question on your answer sheet. 1. To best promote eighth-grade students’ fluency development, a reading teacher should model and assign (A) repeated oral reading of passages at students’ instructional reading levels (B) repeated oral reading of passages at students’ independent reading levels (C) sustained silent reading of passages at students’ instructional reading levels (D) sustained silent reading of passages at students’ independent reading levels

2. Which of the following assignments would best allow sixth graders to demonstrate their understanding of memoirs after a unit covering narrative writing? (A) Each student will craft interview questions, interview an adult, and present the major events of the chosen adult’s life to the class. (B) Each student will use a book read during the unit to create an illustrated timeline of important events in the author’s life. (C) Each student will summarize the common characteristics of memoirs to use as the basis for writing his or her own short memoir. (D) Each student will create his or her own semantic map to demonstrate what was learned about memoirs.

Use the information below to answer the question that follows. -ot pot got lot dot not

-op mop top stop hop

-en hen Ben ten pen

-ar star far car bar tar

3. The word sorting activity above develops students’ understanding of (A) (B) (C) (D)

38

affixes phonemic awareness vocabulary development phonics

Study Guide for the Teaching Reading Test

This ebook was issued to Chelissa Gaines, order #10789211026. Unlawful distribution of this ebook is prohibited.

CHAPTER 7

4. Which of the following approaches would be the best way for a teacher to help parents become involved in their children’s reading achievement? (A) Recruiting parent reading volunteers and asking them to model expressive reading (B) Discussing the school book fairs and the value of family reading nights (C) Explaining reading scores and ways to interact during read-alouds (D) Emphasizing that reading their own book while their child reads his or her own book is a good way to model reading 5. Mr. Martin’s anecdotal notes indicate that a number of his sixth-grade students are struggling readers. To best help these struggling readers, which of the following is the most appropriate next step Mr. Martin should take? (A) Developing after-school tutoring sessions for these students (B) Conducting an assessment to determine the instructional needs of these students (C) Referring these students to the resource teacher to determine if they are eligible for special services (D) Establishing different grading criteria for these students’ assignments 6. Mr. Carlin has noted that many students in his seventh-grade language arts classroom read all genres of assigned text at the same rate. He has determined that student comprehension for expository text in particular is limited, and theorizes that students are reading the text too quickly. Which of the following instructional strategies would be best for Mr. Carlin to use to help his students adjust their reading rate? (A) Designating a specific amount of time students must spend on each reading assignment (B) Giving students study guide questions to answer after they read (C) Using a graphic organizer to guide students in making predictions about what they will be reading (D) Modeling reading different types of texts and explaining when and why readers should slow down

Study Guide for the Teaching Reading Test

7. A fifth-grade teacher facilitates a discussion in which students determine the meaning of a prefix based on a list of student-generated words. Which of the following best describes the objective for this activity? (A) Applying students’ knowledge to decode unfamiliar words (B) Developing students’ ability to apply and transfer knowledge independently (C) Applying students’ knowledge to uncover the meaning of irregular words (D) Developing students’ ability to break words into syllables 8. Which of the following approaches would best help to guide high school students as they conduct peer reviews for writing assignments? (A) Design feedback forms with explicit criteria for discussion and provide time for students to discuss the review (B) Require students to always exchange their writing with the same partner to note longitudinal progress (C) Design feedback forms with areas designated for general commentary and positive reinforcement (D) Require students to create outlines and restate the paper’s main points to confirm that the author’s points were clear 9. A third-grade teacher wants students to develop accountability for improving their literacy skills. The teacher can best promote student responsibility by (A) organizing books according to reading level, genre, and topic (B) requiring students to complete a written response after each day’s reading (C) establishing regular informal assessments to monitor students’ reading progress (D) scheduling weekly interviews to discuss students’ reading goals and progress

39

This ebook was issued to Chelissa Gaines, order #10789211026. Unlawful distribution of this ebook is prohibited.

CHAPTER 7

10. Which of the following practices is most appropriate for a teacher to use when determining the placement of students into flexible groups for reading instruction? (A) Using reading assessments for the students from the previous grade (B) Using students’ current reading levels to form mixed-ability groups (C) Using a current assessment to determine the reading needs of the students (D) Using a teacher-prepared survey to determine the students’ reading interests 11. Which of the following would be the most appropriate informal assessment of a kindergarten student to determine his or her awareness of the concept of print? (A) Asking the student to identify the punctuation at the end of a sentence (B) Providing decodable books for the student to read aloud (C) Asking the student to clap the number of syllables in a set of printed words (D) Providing a word and allowing the student to generate rhyming words 12. The geometry teacher wants students to learn math-specific vocabulary; for example, the names and attributes of the different angles. Which of the following instructional strategies would best help students to retain vocabulary as well as content? (A) Instructing students on the use of morphology to determine the meanings of math words (B) Having students develop semantic charts to list examples of the math words along with similarities and differences between words (C) Providing students with synonyms, antonyms, and homonyms of the math words (D) Having students practice using context clues to determine the meanings of math words

40

13. Which of the following instructional approaches would best help middle school students understand the differences between narrative and expository genres? (A) Analyzing characters through their traits, motivations, and viewpoints (B) Providing writing assignments that emphasize main topics, supporting facts, and organization (C) Analyzing features of text such as chapter titles, table of contexts, and glossaries (D) Providing writing assignments that incorporate illustrations, story maps, and vivid descriptions 14. A high school English teacher observes that struggling readers are unmotivated to participate in any meaningful classroom activity. Which of the following would be the best approach for the teacher to use when planning lessons? (A) Plan student competitions in which students will post their improvements (B) Identify students’ areas of weakness and present material at a challenging level (C) Plan student-directed activities in which students may report their opinions (D) Incorporate relevant material based on students’ interests and strengths 15. Two fifth-grade classes in different cities agree to read the same novel and post their reactions to each chapter in an online community Web site. Each teacher posts thought-provoking questions to begin the online discourse. Which of the following approaches would be best to provide students with feedback about their postings? (A) Providing students with a guideline for what is appropriate and inappropriate to post (B) Acknowledging students’ opinions as accurate and correcting their grammar (C) Tracking the frequency of the students’ postings on the Web site (D) Using a rubric to communicate expectations to students about the quality and frequency of postings

Study Guide for the Teaching Reading Test

This ebook was issued to Chelissa Gaines, order #10789211026. Unlawful distribution of this ebook is prohibited.

CHAPTER 7

16. An eighth-grade English class is beginning a new novel. Which of the following would best help a teacher to determine which words require explicit prereading instruction?

19. A teacher would like to teach students how to create mental images when reading. Which of the following strategies would be the best first step for the teacher to take?

(A) Selecting words from the novel that have common affixes and roots that can be used to determine meanings (B) Identifying words from the novel that are especially challenging because they are figurative or idiomatic (C) Allowing students to identify and compile a list of unfamiliar words they come across in the novel as they read (D) Selecting words that are most relevant for understanding the novel and lack context clues

(A) The teacher can allow the students to teach themselves this process. (B) The teacher can model the strategy by reading a descriptive passage and sharing how he or she creates mental images. (C) The teacher can assign a descriptive reading passage and assess comprehension after students have read the passage. (D) The teacher can have small groups of students discuss mental images that they have formed while reading a passage.

17. To best help fifth-grade students develop metacognitive skills after reading a play with a partner, the teacher should have the partners (A) dramatize various scenes with improvisational skits (B) discuss the skills they applied to help them understand the play (C) reread and summarize their favorite parts of the play (D) evaluate the oral reading skills of their partners using a rubric

20. A kindergarten teacher is setting up a listening center in the classroom. The center activities are most likely to promote reading development by helping the students (A) develop written expression abilities (B) experience a safe, quiet learning environment (C) experience different models of fluent reading (D) strengthen knowledge of elements of fictional text

18. Which of the following is the most valid reason for a teacher to use an alternative to round-robin reading during whole-group instruction? (A) The round-robin consumes valuable instructional time that could be spent on other literacy activities. (B) The round-robin allows opportunities for only the weakest readers in a class to practice oral reading. (C) The round-robin is more effective when used with small groups of students. (D) The round-robin is not as effective in promoting reading skills as compared to silent reading.

Study Guide for the Teaching Reading Test

41

This ebook was issued to Chelissa Gaines, order #10789211026. Unlawful distribution of this ebook is prohibited.

CHAPTER 7

21. A middle school teacher uses the diagram above when teaching vocabulary. Which of the following prompts should be added to the diagram in order to best deepen students’ understanding of and ability to retain a new vocabulary word? (A) (B) (C) (D)

What makes this an important word for me to know? What is the actual dictionary definition of this word? What personal connections can I make to this word? What are other examples of words that share the same root as this word?

22. Which of the following activities would best help students to develop phonemic awareness? (A) The students represent the number of sounds in a given word with chips. (B) The students watch as the teacher reads aloud and points at each word while reading from a big book. (C) The students read a poem written on a chart and then take turns circling words with the short a sound. (D) The students sort word cards into groups based on common rime patterns.

42

23. Tom says things such as, “Her go to a new school now” and “He go to eat last night.” To best help Tom develop proper grammar while speaking, the teacher should (A) have Tom record himself speaking so that he can monitor his improvement (B) follow up with the correct model, such as, “She goes to a new school? Where does she go now?” (C) realize that Tom’s grasp of grammar will naturally become proficient as he matures (D) refer Tom to the school’s speech therapist for a diagnosis

Study Guide for the Teaching Reading Test

This ebook was issued to Chelissa Gaines, order #10789211026. Unlawful distribution of this ebook is prohibited.

CHAPTER 7

24. Whenever John is in the car with his mom and they come to a stop sign, he says, “Mom stop. S-T-O-P.” At school, when the teacher asks John to spell the word “stop,” he cannot. Which of the following methods would best help John make meaningful connections between home and school during spelling instruction? (A) Providing John with reading books about stop signs with environmental print pictures (B) Demonstrating how to sound out words with irregular spelling patterns (C) Creating activities that point out letters and words in environmental print (D) Providing instruction for irregular spelling patterns 25. A first-grade teacher reads aloud a fact-filled children’s story that is set in the rain forest. Which of the following instructional approaches will best help students to connect their prior knowledge to new information learned from the story? (A) Encouraging students to tell stories about their pets (B) Demonstrating note-taking skills to the students (C) Asking students text-to-self questions while reading (D) Emphasizing important details while reading aloud 26. Which of the following statements best describes reading material at a student’s instructional level? (A) The student can decode at least 95% of the words and answer 75% of the passage questions. (B) The student can decode 99% of the words and answer 90% of the passage questions. (C) The student can decode 85% of the words and answer 70% of the passage questions. (D) The student can decode 75% of the words and answer 50% of the passage questions.

Study Guide for the Teaching Reading Test

27. Which of the following procedures, specifically designed to increase sight vocabulary and fluency and accuracy of oral reading, has research shown to be particularly effective? (A) Using a repeated reading program and easy reading materials (B) Using an analytic phonics approach to teaching decoding (C) Introducing all new words using a kinesthetic method (D) Introducing all new words in context and having the students read silently 28. Of the following, which is the best definition of the term “morpheme”? (A) (B) (C) (D)

The smallest sound unit in a language The smallest meaningful unit of a language The grammatical structure within a sentence A meaning clue given by a pause in speech

29. In telling a class that the sound heard at the beginning of the word “dog” is represented by the letter d, a teacher is explaining (A) (B) (C) (D)

morphological analysis semantic relationships syntactic structure phoneme-grapheme correspondence

30. A teacher is concerned that his intermediate-level students use nonstandard English patterns in their speech and writing. A reading specialist who has kept abreast of recent trends would be most likely to advise the teacher to (A) provide extensive practice using written exercises that require students to make choices between standard and nonstandard word forms (B) insist that the students be more consistent in using standard forms in oral communications at school (C) encourage students to be more consistent in using standard forms in oral communications outside of school (D) provide experiences from which the students can conclude that different usage styles are appropriate in different situations

43

This ebook was issued to Chelissa Gaines, order #10789211026. Unlawful distribution of this ebook is prohibited.

CHAPTER 7

31. The words “vain” and “vane” are examples of (A) (B) (C) (D)

antonyms homographs synonyms homophones

32. The ability to distinguish differences or likenesses in individual sounds or words is called (A) (B) (C) (D)

hyperlexia auditory discrimination auditory memory phonological sequencing

33. The best way to develop students’ metacognitive skills is for teachers to do which of the following? (A) Give the students a few global prereading questions to guide their reading. (B) Advocate and model self-questioning during reading. (C) Require that discrete skills be mastered before students begin reading the selection. (D) Provide opportunities for students to write comprehension questions for each other. 34. When introducing new reading vocabulary, a teacher who is aware of the research on language acquisition is most likely to do which of the following? (A) Ensure that the students have the words in their listening and speaking vocabularies (B) Introduce words initially by presenting the isolated sounds and then blending the sounds (C) Have the students learn to spell the words at the time they learn to read them (D) Print the words on the chalkboard and refer to them only when the students ask about them

44

35. Researchers have found that knowledge of word meanings is gained in increments through many experiences. Which of the following is suggested as an effective technique for teaching vocabulary? (A) Assignment of frequent oral presentations (B) Presentation of variations in plot, conflict, and climax in reading (C) Presentation of definitions and other information about words in a lecture format (D) Introduction of new words during lessons in various content areas 36. Some words are most likely to be recognized by elementary students as sight words without using other word study strategies. Examples of such words are (A) (B) (C) (D)

was, a, to, who ray, pay, day, may child, children, some, something sink, sank, sunk, sunken

37. Which of the following is an important feature of the Reading Recovery approach to teaching reading? (A) Explicit phonics instruction is heavily emphasized for all students. (B) New books are selected by students rather than being introduced by the teacher at a slightly more difficult level. (C) Tests are used frequently during the period of instruction to determine students’ levels of reading. (D) Students and teachers interact on a one-onone basis in an individually tailored program.

Study Guide for the Teaching Reading Test

This ebook was issued to Chelissa Gaines, order #10789211026. Unlawful distribution of this ebook is prohibited.

CHAPTER 7

38. As part of the classroom routine, students take turns with daily jobs. This week Sue calls the roll from the attendance sheet, records the number of students absent, and then reads the morning announcements to the class. Pete reads the daily menu aloud and writes it on the board prior to taking lunch count. These routines are most likely to benefit all of the students in their literacy development because they (A) provide the students opportunities for creative responses to reading (B) demonstrate to the students a relationship between real-life situations and literacy (C) offer to the students opportunities to practice discrete skills in isolation (D) help the students to develop home-school connections 39. Which of the following is the best example of an affix? (A) (B) (C) (D)

A prefix or suffix Any syllable in a word A little word in a larger word The root of a word

40. In the word boys, there are how many morphemes? (A) (B) (C) (D)

One Two Three Four

42. Learning to differentiate among words having common roots by looking at the modifications that result when prefixes or suffixes (such as pre-, un-, dis-, -tive, -tion, and -ment) are added to known roots is accomplished through the use of (A) (B) (C) (D)

structural analysis configuration contextual clues phonemic analysis

43. Sam read aloud a 100-word passage about aging pets. He read with five miscues. In one instance, Sam read the sentence below and self-corrected the miscue. Printed sentence: “He was getting old.” Sam initially read: “He was going old.” Sam self-corrected: “He was getting old.” Which of the following conclusions can best be drawn about Sam’s processing strategies from the example above? (A) Sam was probably attending to meaning and was aware that the initial miscue did not make sense. (B) Sam probably did not attend to soundsymbol correspondences because he was familiar with the information. (C) Sam was probably using his schema and background knowledge about aging pets to improve fluency. (D) Sam probably made the mistake because he pays attention only to the beginning and ending sounds of words he does not know.

41. Research shows that children who are allowed and encouraged to “invent spelling” are likely to (A) strengthen their sense of phonemic awareness (B) have difficulty mastering correct spelling (C) wait for someone else to spell words for them (D) be reluctant readers and writers

Study Guide for the Teaching Reading Test

45

This ebook was issued to Chelissa Gaines, order #10789211026. Unlawful distribution of this ebook is prohibited.

CHAPTER 7

44. Structural analysis is the study of meaningful word parts. To use this strategy, students must consider (A) (B) (C) (D)

46. According to schema theorists, vocabulary instruction improves reading comprehension most effectively when students learn

digraphs diphthongs consonant blends inflectional endings

(A) how to use a dictionary or a glossary as an instructional resource (B) a network of relationships among concepts that the word represents (C) how to use context clues as an aid to word recognition (D) to incorporate new vocabulary into written work

45. A science teacher asks the reading specialist for strategies that will help students acquire contentarea vocabulary to improve their comprehension of the science textbook. Which of the following strategies would be most appropriate for the reading specialist to recommend to achieve this goal? (A) Introducing key vocabulary terms in context before reading the text (B) Selecting words from the text and having students copy definitions from the glossary (C) Informing the students that the unit tests will emphasize vocabulary term definitions (D) Pretesting students on the vocabulary terms from the textbook

47. A first-grade teacher uses the running record below to analyze a student’s miscues. Text said: Student said:

mother momma

isn’t ain’t

she went she be goin

cents cent

Based on the miscues, the teacher is most accurate in concluding that the student (A) (B) (C) (D)

46

employs phonics in decoding words reads at the frustration level finds the text very difficult comprehends the text

Study Guide for the Teaching Reading Test

This ebook was issued to Chelissa Gaines, order #10789211026. Unlawful distribution of this ebook is prohibited.

CHAPTER 7

48. What does research suggest is the primary reason why the English language is difficult to decode? (A) The visual-grapheme relationship is generally monolinguistic. (B) The phoneme-grapheme relationship is based on environmental print. (C) The phoneme-grapheme correspondence is based on sight word recognition. (D) The phoneme-grapheme correspondence is inconsistent. 49. A teacher who wants to determine the level of a young child’s phonemic awareness should ask the child to (A) (B) (C) (D)

write her name recite the alphabet identify the rhyming words in a poem identify the sounds in a particular word

50. Which of the following has been shown to correlate most closely with success in learning to read? (A) The student can recite the alphabet. (B) The student has watched little television. (C) The student has the motor skills necessary to hold a pencil and crayon correctly and to use scissors. (D) The student has had extensive experiences with exposure to print. 51. For several days a first-grade teacher has read aloud a poem about birds to the class. Some of the students “read” along with the teacher and followed the tracking of the words as the teacher read aloud. Today the students are asked to say the word bird and feel the place of articulation as they produce the sound at the beginning of the word. They listen and look for words in the poem that begin with this sound and take turns circling the initial b. Which of the following is probably the teacher’s primary reason for using this activity? (A) To expand the students’ knowledge of birds (B) To help the students develop sound-symbol association (C) To develop the students’ vocabulary (D) To enhance the students’ enthusiasm for poetry

Study Guide for the Teaching Reading Test

52. A group of first-grade students is reading a book about a cat. They have written the word cat on lapboards and the teacher asks them to identify a smaller word in it. They identify at, and they are asked to write other words that are in the same word family. Which of the following is probably the teacher’s primary reason for using this activity? (A) To activate and develop the students’ prior knowledge of cats (B) To help the students learn to work in cooperative reading groups (C) To introduce the students to analysis of chunks or patterns in words (D) To emphasize to the students the importance of correct spelling 53. Students in a first-grade class have learned a song and fingerplay about snow. They sing the song and act out the fingerplay over and over until everyone knows it well. The teacher has displayed the words to the song and models how to point to the words as they are said. The students take their turns at tracking the words as they or their classmates chant them. Which of the following is probably the teacher’s primary reason for using this activity? (A) To foster the students’ appreciation of music (B) To help develop students’ awareness of speech/print relationships (C) To assist the students in developing social skills (D) To expand the students’ vocabulary of words associated with snow 54. Which of the following, when encountered during a student’s oral reading, most probably indicates that the student would benefit from reading material at a lower level of difficulty? (A) (B) (C) (D)

Numerous repetitions Immediate corrections Omission of verb endings Meaningful substitutions

47

This ebook was issued to Chelissa Gaines, order #10789211026. Unlawful distribution of this ebook is prohibited.

CHAPTER 7

55. Of the following games, which offers students the greatest opportunity to learn phonics? (A) Students hunt through their reading assignment for words that are examples of meaning units. (B) Students write descriptions of pictures and take turns identifying the pictures from one another’s descriptions. (C) Students sort pictures of objects on the basis of initial sounds, consonant blends or digraphs, rhyming families, or vowel sounds. (D) Students create cloze passages in which they eliminate whole words and have their classmates predict the missing words. 56. Of the following activities, which is most likely to have been inspired by Vygotsky’s theory of the interactive relationship between oral language and thinking? (A) Students read aloud in a round-robin pattern. (B) Students participate in a small-group discussion about a book. (C) A student writes a paragraph that he will then read aloud to the class. (D) A student completes a graphic organizer after reading a passage aloud to herself.

57. After third-grade students had read several selections from Aesop’s Fables, their teacher displayed the following story map.

Story Map Setting: Time

Place

Characters:

Problem:

Action:

Outcome and Moral:

Before the students read another fable about a hare and a tortoise, they looked through the book to get a sense of the story. Guided by questions from the teacher, the students discussed the title and the first paragraphs and illustration. Then they predicted how the new fable could be used to fill in the displayed story map. After reading the fable, the students returned to the map to see if their predictions were changed or verified. The most important benefit of the story-mapping activity was to help the students to (A) improve their comprehension by revisiting the story (B) improve their ability to read the story fluently (C) focus on details of the story line (D) understand the text structure typical of fables

48

Study Guide for the Teaching Reading Test

This ebook was issued to Chelissa Gaines, order #10789211026. Unlawful distribution of this ebook is prohibited.

CHAPTER 7

58. Which of the following is most likely to be included in a language experience approach to teaching reading? (A) Connecting speaking, writing, and reading (B) Memorizing all the letters of the alphabet (C) Reading aloud classic stories from various genres (D) Emphasizing correct letter formation in isolation 59. A sixth-grade teacher has a group of students who read below grade level and seldom read voluntarily outside the classroom. After noticing that these students enjoy talking about movies, the teacher gives them the assignment to choose and independently read a book that has been made into a movie. Then they are to watch the movie together and collaborate to write a comparison of the two versions of the story to share with the class. In making this assignment, the teacher most likely intends to accomplish which of the following? (A) Stimulate the students’ interest in reading (B) Reinforce students’ literal comprehension skills (C) Increase the students’ attention spans (D) Compensate for the students’ reading deficiencies 60. A teacher is introducing a mystery novel to a class as part of the reading curriculum. Which of the following strategies to introduce the book is most likely to be effective in promoting student comprehension? (A) Having the students illustrate a cover for the book (B) Introducing the names of all the characters (C) Reviewing the characteristic elements of the genre (D) Telling the students the solution to the mystery

Study Guide for the Teaching Reading Test

61. A teacher is most likely to ask a student to retell a story or summarize a passage the student has read or listened to in order to assess the student’s (A) (B) (C) (D)

comprehension of the story ability to listen enjoyment of reading vocabulary development

62. Stephan, a first grader, reads, “The frog took toys pulling the sleep” for a text that says, “The friends took turns pulling the sled.” Stephan is overrelying on which of the following? (A) (B) (C) (D)

Syntactic cues Semantic cues Graphophonic cues Structural cues

63. Given the names of students displayed on a name board, early elementary students stretch out the names David, Delano, Cindy, and Adam and listen for the sound of the letter d in each name. This activity is most likely to help students (A) perform morphological analysis (B) increase their awareness of semantic relationships (C) develop their ability to analyze syntactic structure (D) recognize grapheme-phoneme correspondence 64. A third-grade science teacher has noticed that students are having difficulty understanding the assigned readings in the required textbook. Which of the following strategies is likely to be most effective in helping students to better understand what they read? (A) Making a list of vocabulary words for the students to look up and study before reading the text (B) Having the students create a story structure or story grammar of each chapter to aid their understanding (C) Having the students collaboratively create a semantic map associated with a core concept or question (D) Having the students work in pairs and take notes on the text

49

This ebook was issued to Chelissa Gaines, order #10789211026. Unlawful distribution of this ebook is prohibited.

CHAPTER 7

65. A teacher is using reciprocal teaching to encourage students to monitor their own comprehension of a story they are reading in class. Which of the following is the teacher most likely to do initially? (A) Provide a model for students by asking appropriate kinds of questions about the story (B) Present a list of the four types of questions that students are likely to find most helpful (C) Ask students to write down a list of questions about the story (D) Help students to create a graphic illustration of events in the story

50

66. Students are reading a story about two children whose dog has run away. The teacher asks the students to write a journal entry reflecting on how one of the characters feels about what has happened. The teacher is most likely attempting to develop students’ (A) (B) (C) (D)

higher-order thinking skills ability to summarize ability to describe background knowledge

Study Guide for the Teaching Reading Test

This ebook was issued to Chelissa Gaines, order #10789211026. Unlawful distribution of this ebook is prohibited.

CHAPTER 7

Part B 2 Constructed-Response Questions (Suggested time—30 minutes) Directions for Questions 67-68: Answer each constructed-response question on the page designated in the answer sheet booklet. Question 67 Directions: Read the scenario below and then respond to all parts of the task on the pages designated in the answer sheet booklet. The suggested time to spend on this question is 10 minutes. Information about the Student: Bailey, a sixth-grade student, has proficient decoding skills but below-average oral reading fluency skills. Her scores on a recent reading unit test indicate that she struggles with inferring information from the text and predicting viable solutions for problem situations. Bailey scored below average on a timed sight word assessment. When given another list of words and allowed to decode at a slower pace, Bailey was within the average range. The classroom teacher’s anecdotal records indicate that Bailey is reluctant to read in front of her classmates and rarely participates in class discussions. She has difficulty completing assignments independently and often asks the teacher for assistance. Task: 1)

Based on the information provided above, identify and describe one reading difficulty that Bailey is experiencing. Support your choice by including specific details about Bailey’s observed behavior.

2)

Describe one instructional strategy, for the reading difficulty identified in task 1, that Bailey’s teacher could use to help Bailey improve her reading abilities. Be sure to explain how the strategy would benefit Bailey by including specific references to Bailey’s reading behaviors.

3)

For the instructional strategy you identified in task 2, describe one instructional material or technology component that Bailey’s teacher could use to support her instruction. Provide a rationale for using the instructional material or technology component that you have described.Your rationale must include evidence of your understanding of the reading process.

Study Guide for the Teaching Reading Test

51

This ebook was issued to Chelissa Gaines, order #10789211026. Unlawful distribution of this ebook is prohibited.

CHAPTER 7

Question 68 Directions: Read the scenario below and then respond to all parts of the task on the pages designated in the answer sheet booklet. The suggested time to spend on this question is 10 minutes. Scenario: A teacher received the following writing from a fourth grade student, Diana: How to tack car of a cut kittin the things you will be needing are cut toys 2 smal bools a big cage scraching post kitty litte and scopr first you will be needing a ruff scraching post because they will scrach your frnchr and put holls in it next you will be needing tow smal food bools one is for food the other one is for kool woter finlee you will be needing cut toys because toys muck thum actave so the kittin dose not get angre thet is how to tack car of a cut kittin. Task: 1)

Based on the information provided above, identify one area that is affecting Diana’s ability to communicate effectively in writing. Include specific details about Diana’s writing ability as portrayed in the writing sample above.

2)

For the area identified in Task 1, describe one instructional strategy that Diana’s teacher can use to help Diana improve her ability to communicate effectively in writing. Be sure to explain how this strategy would benefit Diana by including specific references to Diana’s writing sample.

3)

Identify and describe one specific way to assess Diana’s progress during the year. Be sure to include in your description how the assessment will be utilized and how the data collected will impact Diana’s future instruction.

52

Study Guide for the Teaching Reading Test

This ebook was issued to Chelissa Gaines, order #10789211026. Unlawful distribution of this ebook is prohibited.

Chapter 8

Right Answers and Explanations

























This ebook was issued to Chelissa Gaines, order #10789211026. Unlawful distribution of this ebook is prohibited.

CHAPTER 8

Now that you have answered all of the practice questions, you can check your work. Compare your answers to the multiple-choice questions with the correct answers in the table below. Question Number

Correct Answer



1 2 3

B C D



4 5

C B



6 7

D B



8 9 10 11 12 13 14 15 16 17 18 19 20 21

A D C A B C D D D B A B C C



22 23 24 25 26 27 28 29 30 31 32 33 34 35

A B C C A A B D D D B B A D

54

Content Category

Comprehension and Fluency Comprehension and Fluency Alphabetic Principle/Phonics and Word Analysis Emergent Literacy Alphabetic Principle/Phonics and Word Analysis Comprehension and Fluency Alphabetic Principle/Phonics and Word Analysis Comprehension and Fluency Comprehension and Fluency Comprehension and Fluency Emergent Literacy Vocabulary Comprehension and Fluency Comprehension and Fluency Comprehension and Fluency Vocabulary Comprehension and Fluency Comprehension and Fluency Emergent Literacy Comprehension and Fluency Alphabetic Principle/Phonics and Word Analysis Phonological Awareness Emergent Literacy Emergent Literacy Comprehension and Fluency Alphabetic Principle/Phonics and Word Analysis Comprehension and Fluency Alphabetic Principle/Phonics and Word Analysis Phonics Emergent Literacy Vocabulary Phonological Awareness Comprehension and Fluency Vocabulary Vocabulary

Question Number



Correct Answer



36

A



37 38 39 40 41 42 43 44

D B A B A A A D



45 46 47 48 49 50 51 52

A B D D D D B C



53 54

B A



55 56 57 58 59 60 61 62

C B C A A C A C



63 64 65 66

D C A A

Content Category

A lphabetic Principle/Phonics and Word Analysis Alphabetic Principle/Phonics and Word Analysis Comprehension and Fluency Word Analysis Alphabetic Principle/Phonics and Word Analysis Phonological Awareness Vocabulary Comprehension and Fluency Alphabetic Principle/Phonics and Word Analysis Vocabulary Comprehension and Fluency Comprehension and Fluency Emergent Literacy Phonological Awareness Emergent Literacy Alphabetic Principle/Phonics and Word Analysis Alphabetic Principle/Phonics and Word Analysis Emergent Literacy Alphabetic Principle/Phonics and Word Analysis Alphabetic Principle/Phonics and Word Analysis Emergent Literacy Comprehension and Fluency Emergent Literacy Comprehension and Fluency Comprehension and Fluency Comprehension and Fluency Alphabetic Principle/Phonics and Word Analysis Alphabetic Principle/Phonics and Word Analysis Comprehension and Fluency Comprehension and Fluency Comprehension and Fluency

Study Guide for the Teaching Reading Test

This ebook was issued to Chelissa Gaines, order #10789211026. Unlawful distribution of this ebook is prohibited.

CHAPTER 8

Explanations of Right Answers 1. This question asks you to apply your knowledge of fluency development practices. (A) is not the correct answer because fluency development should occur at students’ independent reading levels. (C) and (D) are incorrect because research has not proven that independent reading on its own helps to develop a student’s fluency. The correct answer is (B). Repeated reading is an effective way to increase a student’s reading fluency. 2. This question asks you to apply your knowledge of comprehension, specifically the use of the reading-writing connection to develop students’ understanding of a genre. (A) is incorrect. This assignment is a good prewriting activity because it establishes significant questions. However, it neglects the fact that a memoir is autobiographical. (B) is incorrect because creating a timeline of a previously read memoir will not demonstrate students’ understanding that memoirs include stories that tell of a life lesson. (D) is not correct because semantic maps will show declarative knowledge about the genre but will not demonstrate a procedural knowledge that demonstrates they are able to write memoirs. (C) is the best answer. Creating a list of characteristics as a class will provide an informal assessment of the students’ understanding of memoirs and their ability to generalize from examples. Also, writing a memoir will show a level of understanding of the characteristics of the genre.

Study Guide for the Teaching Reading Test

3. This question asks you to apply your understanding of word families. (A) is incorrect because affixes are morphemes that are added to the endings of words. Affixes change a word’s meaning. Some examples of affixes include “-ing,” “-ment,” “-ed,” and “-s.” (B) is incorrect. The sorting activity involves analyzing the relationship between print letters and their corresponding sounds. Phonemic awareness refers to the knowledge that spoken words are made up of smaller parts. (C) is also incorrect. The sorting activity does not draw on or increase a student’s knowledge of what the words in the sort mean. Rather, it focuses on looking and listening for similar patterns (rimes) in the words. Only phonics accurately describes the skill being practiced during this activity, because phonics is associating sounds with letters to make words and looking for common patterns to read new words. Therefore, the best answer is (D). 4. This question asks you to apply your understanding of emergent literacy. (A) is not correct. For various reasons, not all parents would be able to participate in being a parent volunteer. (B) is not correct. School book fairs and family reading nights would not necessarily teach parents the way to interact with their child to affect their child’s reading progress. (D) is not correct because modeling reading to a child is important, but it does not involve the parent and student interacting so the student may only appear to be reading. (C) is the best answer. Providing information to the parents and explaining ways to read and interact with their child would be most beneficial.

55

This ebook was issued to Chelissa Gaines, order #10789211026. Unlawful distribution of this ebook is prohibited.

CHAPTER 8

5. This question asks you to apply your knowledge of addressing varying reading needs of students in a classroom. (A) is incorrect. Tutoring sessions without assessing the student’s needs would not be the most beneficial and effective instructional practice. (C) is not correct. Several instructional interventions should be implemented and documented before being referred to a specialist. (D) is also incorrect. Changing the ways in which students are graded does not identify their academic needs. The best answer is (B). Assessing students’ instructional needs and noting their skill strengths will allow teachers to first build the students’ confidence. Then the teacher can create flexible grouping so that students who need work on the same skills may begin acquiring them through practice.

8. This question asks you to use your knowledge about giving useful feedback to students. (B) is incorrect. Students could benefit from hearing different perspectives of their writing. In addition, as a student reviews other students’ writing styles, they may discover the advantages and disadvantages of different writing styles. (C) is incorrect. A feedback form that reminds students to write positive comments is only useful if there are clear instructions about what types of comments the writer is able to address. (D) is also incorrect. Restating the paper’s main points would be a specific type of review but would not serve as a general peer review for various writing purposes. (A) is the best answer. The explicit criterion provides students with a guide for reviewing other students’ writing and suggestions for improving their own.

6. This question asks you to apply your knowledge of fluency. (A) is incorrect because setting a time will not assist students in fluency. (B) is incorrect because study guide questions will help increase comprehension, not fluency. (C) is also incorrect. Like study guide questions, graphic organizers will assist in comprehension, not fluency. (D) is the best answer. Modeling reading for students will be the best assistance to improve fluency.

9. This question asks you to apply your knowledge of informal reading assessment practices. (A) is incorrect. This is not holding students accountable for reading during the time provided regardless of whether the books were well selected or not. (B) is incorrect. Planning for the students does not generate self-monitoring skills. However, in guided reading choosing books to develop specific skills is beneficial. (C) is incorrect. Informal assessments are a good way to see reading progress; however, this is information that will guide the teacher’s instruction rather than require student accountability. (D) is the best answer. Meeting with students individually for a short time to discuss how much and what they want to read by a certain date will help the students monitor their own reading habits and assure the teacher that students can hold themselves accountable for their own learning.

7. This question asks you to demonstrate your understanding of word analysis. (A) is incorrect. The term “decode” in reading is to apply knowledge of letter-sound relationships in order to pronounce words. (C) is incorrect. Students are not looking for a strategy to understand irregular words. Rather, they are looking for words that have prefixes in common and looking for common meanings. (D) is incorrect. Phonemic awareness is the ability to identify and manipulate large units of oral language—words, syllables, onsets, rimes, phonemes. (B), therefore, is the best answer. If students are led through the thought process of comparing new words with words they already know, then they will have a method for determining meaning.

56

Study Guide for the Teaching Reading Test

This ebook was issued to Chelissa Gaines, order #10789211026. Unlawful distribution of this ebook is prohibited.

CHAPTER 8

10. This question asks you to demonstrate knowledge of effective grouping strategies. (A) is not correct because flexible groups are intended to improve skills for students and would need more current information about the students’ abilities for specific reading skills. (B) is not correct. Overall reading levels are not the criteria utilized for flexible groups but are helpful if planning other small-group activities. However, mixed-ability grouping is not appropriate for reading group instruction. (D) is also not correct. Common reading interests are not a way to determine small flexible groups. Therefore, (C) is the best answer. Current assessments of students’ reading skills in different areas of reading (decoding, fluency, summarization, etc.) would help teachers prepare their flexible groupings. 11. This question asks you to show your understanding of emergent literacy, specifically concepts of print. (B) is incorrect. Reading decodable materials is a better indicator of proficiency in letter-sound recognition. (C) is incorrect. Identifying syllables in printed words would indicate phonological awareness. (D) is incorrect. Creating rhymes is an indicator of phonemic awareness. (A) is the best answer because students who are able to identify the ending of a sentence have an awareness that print is made up of sentences. 12. This question asks you to demonstrate your knowledge of effective ways to teach contentarea vocabulary. (A) is not the correct answer. Studying the structure of words in content areas often does not help students with word meaning. (C) is not correct. A lot of contentarea words do not have a synonym, antonym, or homophone that students can associate that word with. (D) is not correct because context clues in textbooks are typically not clear and concise enough for students to understand. (B) is the best answer because a semantic chart lists examples, features, and definitions, giving students different strategies for vocabulary acquisition.

Study Guide for the Teaching Reading Test

13. This question asks you to demonstrate your knowledge of comprehension. (A) is not correct. Character analysis will help students understand the features of narratives but would not be broad enough for expository works. (B) is incorrect because the writing features described will address expository works but not narratives. (D) is also incorrect. These features are mainly seen in narratives but do not address features of expository writing. (C) is the best answer because features of text will be a good overarching method to identify differences between narratives and expository works. 14. This question asks about the role of motivation in reading achievement. (A) is incorrect because students who are struggling in reading are not confident in their own skills. For this reason, posting their results or even comparing their progress with others can actually be counterproductive. (B) is incorrect. To motivate students you would want to build from their strengths so they will develop confidence, and then add challenging lessons at an appropriate level. (C) is incorrect. If students are struggling, they are in need of direct, explicit instruction. (D) is the best answer. Students who are struggling would be motivated by beginning with activities that they are good at and are interested in. Then as they build confidence and feel connected, or believe the lessons are relevant to their lives, they will continue to be engaged in learning.

57

This ebook was issued to Chelissa Gaines, order #10789211026. Unlawful distribution of this ebook is prohibited.

CHAPTER 8

15. This question asks you to apply your knowledge of assessment of readers’ responses. (A) is not the correct answer. Providing guidelines regarding appropriateness of postings is a document a teacher should prepare and discuss prior to beginning the online community. (B) is not the correct answer. The students’ opinions are a result of their own interpretations, and commenting on their accuracy and grammar would discourage them from posting future comments. (C) is not correct. The purpose of the site is to promote critical thinking, such as a classroom discussion. Therefore making it a structured grade may be counterproductive in this scenario. (D) is the best answer. Addressing each student’s effort will acknowledge where the student is in their writing abilities and noting improvements will encourage students. Given this site’s purpose is to promote critical thought, positive feedback will serve to inspire, not stifle, their thoughts. 16. This question asks you to apply your knowledge of effective vocabulary instruction. (A) is incorrect. This would not encompass enough types of vocabulary words. (B) is incorrect because these would be good but would not encompass enough vocabulary words. (C) is not correct. This is a selfmonitoring strategy to be used during reading and would not allow for the prereading activity to aid students’ comprehension. (D) is the best answer. If the context allows for students to make sense of the word and if there is enough information to allow students to connect the word to their background knowledge, no additional instruction is needed. Words that are critical to understanding the text and cannot be determined through contextual clues will need explicit instruction.

58

17. This question asks you to apply your understanding of the role of metacognition in comprehension. (A) is not correct. Thinking on the spot to improvise skits is a good skill but is not metacognitive. (C) is not correct. Rereading the play if they did not memorize the lines correctly is not metacognitive—thinking about thinking—during reading. It is a check of the correct wording. (D) is also not correct. Discussing students’ oral reading skills is not thinking about thinking but is a helpful strategy for fluency. However, directing students to discuss with other students the strategies that help them understand text is metacognitive. Therefore, (B) is the best answer. 18. This question asks you to demonstrate your knowledge of effective oral reading strategies. (B) is incorrect. Round robin reading can cause unnecessary subvocalization. While one reader is reading aloud, the others are expected to follow along, reading silently. Because oral reading is slower than silent reading, the silent readers are therefore encouraged to subvocalize every word. This subvocalization may become internalized and cause slower reading rates. (C) is incorrect. Round robin reading is unrehearsed and can be challenging and frustrating especially because teachers correct students’ errors in a public way, usually before the students can attempt to self-correct. In addition, oral reading, being much slower than silent reading, takes longer; the number of words that students will read over a school year can actually decrease. (D) is also incorrect. Round robin reading can hamper listening comprehension. Instead of truly listening to others read, students are preoccupied with following lines of print and looking ahead, either because they are bored or because they are trying to give themselves some practice before they will be expected to read aloud before others. Therefore, (A) is the best answer. Round robin reading consumes valuable classroom time that could be spent on other meaningful activities. Because oral reading, being much slower than silent reading, takes longer, the number of words that students will read over a school year can actually decrease.

Study Guide for the Teaching Reading Test

This ebook was issued to Chelissa Gaines, order #10789211026. Unlawful distribution of this ebook is prohibited.

CHAPTER 8

19. This question asks you to show your understanding of teaching of comprehension strategies. (A) is not correct because students cannot teach themselves the process of creating mental images. (C) is not correct. This practice does not ensure that the students have formed mental images. (D) is not correct. This also does not ensure effective teaching of the strategy. (B) is the best answer. Modeling the practice is very effective in showing students what is expected. 20. This question asks you to show your understanding of fluency. (A) is incorrect because this would not be age appropriate for kindergarten students. (B) is incorrect because a quiet, safe learning environment may or may not be what the students experience. A print-rich environment would promote reading development. (D) is incorrect. The center could provide both fiction and nonfiction texts. Simply listening to a book does not necessarily lead to an increased knowledge of story elements without any teacher guidance or instruction. (C) is the best answer. Audiotapes allow students to follow the print in the book and to hear different models of fluency. The young child may not be able to read this material by themselves. Listening to the audiotapes provides a model for fluency that the child may then mimic. 21. This question asks you to show knowledge of vocabulary instruction. (A) is incorrect because it is based on the student’s opinion of a word, not its actual importance to the concept being taught. (B) is incorrect. Looking up definitions does not ensure student understanding of terms. (D) is also incorrect. Asking for other words is not an appropriate strategy for learning vocabulary words. Therefore, (C) is the best answer. Making personal connections to a word will help students see how the word is related to their everyday lives, therefore making the vocabulary word more relevant and easier to recall. This prompt also asks students to gain deeper understanding of the word because they must determine how the word applies to their prior knowledge.

Study Guide for the Teaching Reading Test

22. This question asks you to apply your knowledge of phonemic awareness. (B) is not correct. Pointing at each word as the teacher reads aloud from a big book is a practice used to promote speech-text connection and concept of word. (C) and (D) are not correct because both of these activities target phonics skills, not phonemic awareness. These activities involve matching sounds to printed letters. The best answer is (A). Phonemic awareness is the awareness that spoken language is made up of individual sounds, which can be demonstrated by representing the number of sounds in a word with chips. 23. This question requires you to demonstrate knowledge of oral language. (A) is not the correct answer. He would be able to monitor his own improvements; however, a critical aspect is to receive correction. (C) is not the correct answer. He would be writing incorrect grammar and confuse himself more. (D) is not the correct answer. Speech therapy focuses on pronunciation and articulation. (B) is the best answer. He needs proper correction and proper modeling to practice proper grammar daily and in all environments. 24. This question is about emergent literacy. (A) is incorrect. This could help, but only focuses on one word. (B) is incorrect. This is a traditional method that neglects the importance of establishing home-school connections; these connections make school information personally relevant. (D) is also incorrect. Again, this is a traditional method of approaching spelling but does not address the concern to make connections between home and school. (C) is correct. Activities that point out print concepts in familiar environmental images will bring literacy learning to the students’ home lives.

59

This ebook was issued to Chelissa Gaines, order #10789211026. Unlawful distribution of this ebook is prohibited.

CHAPTER 8

25. This question asks you to apply your knowledge of encouraging the development of comprehension strategies. (A) is incorrect. Stories about pets may or may not be relevant. (B) is incorrect because note-taking skills in this context are not age appropriate. (D) is incorrect. Emphasizing details may not reach the goals of bridging prior knowledge with new knowledge. (C) is the best answer. Pausing to discuss the story and making connections is a good way for students to incorporate their own experiences with new information learned. 26. This question asks you to apply your knowledge of reading levels. (B) is incorrect. The student can decode the words but cannot answer half of the passage questions. This may indicate a problem in decoding words and not comprehending what the student reads. (C) is incorrect. If the student can only decode half of the words, the passage is not at the correct level for a student. Therefore, the text is at a frustration level for a student. This will decrease the student’s motivation to read. (D) is incorrect. The student is presently reading at the frustration level. The best answer, therefore, is (A). For the text to be at the student’s instructional level, the student should be able to read 95% of the words and answer correctly at least 75% of the passage questions. 27. This question asks you to demonstrate your knowledge of fluency instructional practices. (B) is incorrect because using an analytical approach to teaching decoding will not help a student to recognize words automatically. (C) is incorrect. Cloze exercises will show if students understand how to use new vocabulary words but will not improve fluency. (D) is also incorrect. Reading silently on its own will not necessarily improve oral reading fluency and automaticity of word recognition. Therefore, (A) is the best answer. Using reading materials at a student’s independent level and doing repeated readings allows students to gain familiarity with the text. This leads to automatic recognition of sight words, increased accuracy, and increased fluency in oral reading.

60

28. This question asks you to apply your knowledge of word analysis. (A) is incorrect because it describes a phoneme. (C) is incorrect because it refers to the syntax of a sentence. (D) is incorrect because the stressed element in a word is an accented syllable. A morpheme is the smallest meaningful unit of a language, making (B) the correct answer. 29. This question asks you to show your knowledge of phonics. (A) is incorrect. Morphological analysis deals with units, such as prefixes, within words that carry meaning. (B) is incorrect. Semantic relationships refer to the relationship in meaning between words, not related to phonics. (C) is incorrect. Syntactic structure deals with the grammatical aspect of a sentence and the way word order creates meaning. (D) is the best answer. Matching a sound to a corresponding letter that represents a sound is referred to as phoneme-grapheme correspondence. 30. This question asks you to demonstrate your knowledge of oral language. (A) is not correct. Choosing between the standard form and nonstandard form of English on a worksheet will most likely not change students’ actual speech or writing patterns. (B) is not correct. Increasing the amount of writing that students must do will not, by itself, lead to standard usage of English. (C) is not correct. It would be impossible and ineffective to monitor students’ use of standard English outside the classroom. Therefore, (D) is the best answer. Providing purposeful experiences for the students so they can see the appropriateness of language in different settings makes a difference. 31. This question asks you to apply your knowledge of vocabulary. (A) is incorrect because antonyms are words that have opposite meanings. (B) is incorrect because homographs are words that look the same but have different meanings, such as “wind” and “wind” (pronounced like “kind”). (C) is also incorrect. Synonyms are words that have similar meanings. (D) is the best answer. Homophones are words that sound the same but are spelled differently and have different meanings.

Study Guide for the Teaching Reading Test

This ebook was issued to Chelissa Gaines, order #10789211026. Unlawful distribution of this ebook is prohibited.

CHAPTER 8

32. This question asks you to show your knowledge of phonemic awareness. (A) is incorrect. Hyperlexia is a condition in which the main characteristics are an above-normal ability to read accompanied by a below-normal ability to understand spoken language. (C) is incorrect. Auditory blending is orally putting individual sounds together to say a whole word. (D) is incorrect. Phonological sequencing refers to the sequence of sounds in a word. (B), therefore, is the best answer. The ability to distinguish differences and similarities between individual sounds or words is the definition of auditory discrimination. 33. This question asks you to apply your knowledge of comprehension. Metacognition is the ability to think about your thinking. It is a process where students monitor their own learning. (A) is not the correct answer. Giving prereading questions to the students will not develop their metacognitive skills. (C) is not the correct answer. Having students memorize new vocabulary may increase their comprehension of the whole selection but will not develop students’ metacognitive skills. (D) is not the correct answer. (B) is the best answer. Modeling how to self-question while reading is the best way to guide students to practice and internalize metacognitive skills.

Study Guide for the Teaching Reading Test

34. This question requires you to apply your knowledge of vocabulary acquisition. (B) is incorrect. Isolating and then blending the sounds is not the best way to introduce new reading vocabulary. This approach will not help the students to learn the meaning of the vocabulary or give examples of its usage so that they understand the word when they come across it in their reading. (C) is incorrect. Learning how to spell newly introduced vocabulary words does not promote understanding of the words’ meanings. (D) is also incorrect. Referring to the words only when students ask is not an effective strategy. (A) is the best answer. Many and varied firsthand experiences with language attached to them help students develop vocabulary. These provide opportunities for them to test and practice these new words against their actual referents and to receive direct and concrete feedback, often in the form of conversations. Later, the words in a passage help students to reconstruct the author’s meaning from the students’ own experiences. 35. This question asks you to apply your knowledge of vocabulary acquisition. (A) is incorrect. Assigning frequent oral presentations is not an effective technique for teaching vocabulary since there is no teacher guidance involved. (B) is incorrect. Presenting variations in plot, conflict, and climax does not address vocabulary. (C) is also incorrect. This answer choice does address vocabulary, but a lecture is not the most effective way to give students the needed multiple exposures for them to internalize vocabulary. (D) is the best answer. Research shows that children learn words more quickly and remember words longer when they are embedded in meaningful contexts rather than in lists or in isolation.

61

This ebook was issued to Chelissa Gaines, order #10789211026. Unlawful distribution of this ebook is prohibited.

CHAPTER 8

36. This question requires you to demonstrate your knowledge of sight words. A sight word is a word that is immediately recognized as a whole and does not require analysis for identification. It is taught as a whole word. In option (B) all are examples of word families, which can be decoded. (C) includes examples of words with similar roots. (D) includes the same words in different verb tenses. (A) is the best answer. This is the only option that includes four examples of sight words. 37. This question asks you to demonstrate knowledge of practices for struggling readers. (A), (B), and (C) are not true in a Reading Recovery® program. Each child’s strengths and needs will be unique and, depending upon instructional experiences, may or may not need phonics instruction. (D) is the best answer. Children reread several self-selected books from their individual book boxes daily, but the new book is selected by the teacher to offer a slightly more difficult challenge. The teacher keeps a running record and observes the child’s use of strategies during reading and writing. 38. This question tests your knowledge of how to make reading purposeful and promote a positive reading environment. (A) promotes the connection between reading and writing, which does not apply to the activities described within the question. (C) is incorrect. In (C) discrete skills are clear and well-defined actions that have a beginning and an end, such as hitting or throwing; these skills do not apply to this question. (D) states that developing a good home-school connection is critical, but establishing this connection was not performed in class. (B) is the best answer. Research shows that the best way to develop literacy skills is to have meaningful opportunities to practice them.

40. This question asks you to show your knowledge of word analysis. (A), (C), and (D) are not correct because they do not list the correct number of morphemes. A morpheme is the smallest meaningful part or unit of a word. The word “boys” has two morphemes: the base “boy” and the ending “s.” Therefore, (B) is the best answer. 41. This question assesses your knowledge of the connection between phonemic awareness and invented spelling. (B) is incorrect. Because spelling is a developmental process, students will have an easier time spelling when using invented spelling. (C) is not an appropriate strategy. Students can become dependent on others for spelling needs and may therefore not try to spell, which will hinder their spelling progress. (D) is also incorrect. Students are more apt to want to read and write when they can try on their own and know that it’s okay to write the sounds they hear. (A) is the best answer. Research supports the connection between invented spelling and phonemic awareness. Invented spelling helps children learn to spell as well as to use phonics as a way of analyzing words. 42. This question is testing your knowledge of vocabulary development. (B) is incorrect. Configuration refers to the arrangement of parts or elements. (C) is incorrect. Contextual clues are hints an author provides throughout a passage to define words. (D) is also incorrect. Phonemic analysis refers to the process of segmenting words. The best answer is (A). Structural analysis is the study of meaningful word parts such as roots, prefixes, and suffixes.

39. This question asks you to demonstrate your knowledge of terminology related to word analysis. Answer choices (B), (C), and (D) are parts of words, but not affixes. (A) is the best answer. An affix is a syllable or group of syllables that when added to a base word or a root word alters the meaning.

62

Study Guide for the Teaching Reading Test

This ebook was issued to Chelissa Gaines, order #10789211026. Unlawful distribution of this ebook is prohibited.

CHAPTER 8

43. This question assesses your ability to analyze assessment data. (B) does not support the fact that Sam self-corrects, knowing the sentence did not sound correct as he read it the first time. (C) is incorrect. Sam was able to identify his mistake because of his reading self-monitoring skills, not fluency. (D) is also incorrect. He discovered his mistakes based on what he heard himself read, not on his decoding skills. (A) is the correct answer. The goal in reading is the construction of meaning. Once the reader altered the textual meaning intended by the writer, the reader’s self-correction demonstrated his integration of all three cuing systems.

47. This question asks that you demonstrate your ability to analyze reading assessment data. (A) is incorrect. The student’s miscues show that the student is not using phonics to sound out words (i.e., the student reads “be goin” when the text is “went”). The student is using semantic cues to read the text. (B) and (C) are incorrect. The student’s miscues do not take away from the meaning of the text. Therefore the student is not reading at a frustration level, nor is the student finding the text difficult. (D) is the best answer. The student comprehends the text as evidenced by the student’s translation of language to dialect.

44. This question assesses your knowledge of phonics skills. (A) is incorrect. Diagraphs are two consonants representing one sound. (B) and (C) are also incorrect. Diphthongs are two vowels as one syllable, and consonant blends are a group of consonants with no intervening vowel. The best answer is (D). Inflectional endings are the modification of a word to change meaning, which directly relates to structural analysis.

48. This question is assessing your knowledge of the language acquisition process. (A), (B), and (C) are incorrect. It is not due to a visualgrapheme relationship, environmental print, or sight word recognition. (D) is the best answer. The English language is difficult to decode because the grammatical rules are inconsistent, making it hard for students who are learning English. This is due to the phoneme (smallest unit of sound)-grapheme (a letter representing a phoneme) relationship not always being the same or following a consistent pattern.

45. This question tests your knowledge of vocabulary acquisition in the content areas. (B) is not an effective strategy for learning new words. (C) is essential so students know what the expectations are, but does not ensure vocabulary acquisition. (D) is also appropriate to determine the student’s background knowledge. However, it is critical to follow up with a vocabulary-in-context introductory lesson. (A) is the best answer choice. Introducing a new key term in context will aid students with the application process and integrating it into their speaking vocabulary. 46. This question asks you to demonstrate knowledge of schema theory, a theory important to reading comprehension. All the other strategies mentioned in (A), (C), and (D) are effective for vocabulary acquisition but do not relate to schema theory. People organize information in the brain by connecting new information with what they already know. Out of all the answer choices, (B) is the only one that promotes connecting new learning with previously known information or concepts.

Study Guide for the Teaching Reading Test

49. This question tests your understanding of phonemic awareness. (A) is incorrect. Having a student writing his or her name will help the teacher determine letter-writing skills and memory. (B) is incorrect. Reciting the alphabet is related to a letter-name fluency skill, not phonemic awareness. (C) is a phonological awareness skill, not part of phonemic awareness which is what distinguishes the two. (D) is the best answer. Phonemic awareness refers to the ability to hear, manipulate, and identify sounds in spoken words. 50. This question evaluates your knowledge about the language acquisition process. (A), (B), and (C) are incorrect. Reading success is not indicated by reciting the alphabet, watching little television, or having developmentally appropriate motor skills. However, they give the teacher good background information about a student. (D) is the best answer. Exposure and extensive experiences with text are proven to be effective indicators of learning success.

63

This ebook was issued to Chelissa Gaines, order #10789211026. Unlawful distribution of this ebook is prohibited.

CHAPTER 8

51. This question addresses the reading component known as alphabetic principle. Although (A), (C), and (D) are effective strategies, they are not related to the activity described in the question. (B) is the best answer. Dividing words into onsets and rimes is easier and more natural for children than dividing them into syllables. What comes before the vowel is the onset; the vowel and what follows is the rime. 52. This question asks you to demonstrate your knowledge of phonics. The activity described in the question is not intended to activate background knowledge (A), promote cooperative learning groups (B), or emphasize the importance of proper spelling (D). The best answer is (C). Working with rimes or word families promotes reading by allowing students to analyze a particular pattern in a word and identify that pattern in other words they know. 53. This question assesses your knowledge of instructional strategies for emergent readers. Although all the answer choices, (A), (C), and (D), are effective strategies teachers can utilize in their classrooms, (B) is the only one that addresses the teacher’s objective as described within the question. With the repetition, modeling, and tracking of the finger play, this teacher is helping students to connect speech to print. 54. This question asks about oral reading miscues. (B) and (D) are incorrect. Making immediate self-corrections and meaningful substitutions indicates that the reading material is at the correct level for the student. (C) is also incorrect. Leaving off verb endings does not indicate that the reading material should be at a lower level of difficulty. (A) is the best answer. Numerous repetitions can indicate that the reader is having difficulty applying word identification skills to the next word or group of words. In this case, the reading material is too hard for the reader.

64

55. This question is assessing your knowledge of phonics. Although all the answer choices include effective strategies, only one offers students the greatest opportunity to learn phonic skills. Answer choices (A), (B), and (D) are incorrect because they all help students build their vocabulary skills. (C) is the best answer because sorting pictures by initial sounds, etc., gives students a chance to apply their knowledge of phonetic patterns and skills. 56. This question asks about oral language. Vygotsky’s theory of the interactive relationship between oral language and thinking talks about how children’s private speech allows them to develop strategies and plan activities that ultimately aid their development. Answer choices (A), (C), and (D) do not promote a connection between thinking and social interaction with others. (B) is the best answer. Of all the answer choices, the only clear choice that integrates thinking with oral language is (B). 57. This question asks about comprehension. (A) is incorrect because the students did not revisit the story to complete the story map. (B) is incorrect. Increased reading fluency is not a direct outcome of a story-mapping activity. (D) is also incorrect. Understanding the text structure that is typical of fables is not the most important benefit of the story-mapping activity. In addition, the elements of a story map can be found in other genres of fiction and are not limited to fables. (C) is the best answer. The main purpose of the story-mapping activity is to focus students’ attention on details of the story line. The students used prior knowledge to make predictions about the probable setting, problem, and solution of the story. After reading, students focused once again on the details of the story line in order to compare their predictions to what actually happened in the story.

Study Guide for the Teaching Reading Test

This ebook was issued to Chelissa Gaines, order #10789211026. Unlawful distribution of this ebook is prohibited.

CHAPTER 8

58. This question asks about common instructional practices in reading. Answer choices (B), (C), and (D) are not strategies related to LEA and are, therefore, incorrect. The best answer is (A). The Language Experience Approach (LEA) is an instructional reading method that uses orally dictated experiences of the student as a text. Those experiences are written down by the teacher, and read and illustrated by the student. (A) is the only answer choice that includes activities related to this method. 59. This question is testing your knowledge of motivational reading strategies for struggling readers. (B) is incorrect. Literal comprehension is one of the lower-order skills in Bloom’s taxonomy, and comparing a movie with a book is a higher-order skill. (C) is also incorrect. This activity is not proven to increase a student’s attention span. (D) is incorrect. This particular strategy does not address reading deficiencies; it only increases a student’s motivation to read. (A) is the best answer. Reading independently and increasing student interest in reading were the main goals of the assignment created by the teacher. By connecting their personal interests with reading, the teacher made reading fun and purposeful. 60. This question is assessing your knowledge of reading comprehension. (A) is an ineffective strategy because designing a cover for a book that has not been read does not show understanding of the story plot. (B) is also incorrect. Introducing the names of the characters does not guarantee comprehension of the story. (D) is incorrect because by telling the students the solution to a mystery, they will no longer need to think about what they read, thus not improving comprehension of the text. The best answer is (C). Reviewing the characteristic elements of mysteries is a strategy proven to help students understand what they read. Research shows that frequent use and response to print correlate most closely with success in learning to read.

Study Guide for the Teaching Reading Test

61. This question evaluates your knowledge of reading assessment. (B) is not correct. Retellings often occur after students silently read a text so the ability to listen will not necessarily be assessed. (C) is not correct. Accurately retelling a story is not an indication of a student’s level of enjoyment of reading. (D) is not correct. A retelling is not used to assess a student’s vocabulary development. (A) is the best answer. A retelling is a postreading assessment in which readers or listeners tell what they remember either orally or in writing. Teachers can assess a student’s comprehension of a story or passage through a student’s retelling. 62. This question asks you to apply your ability to draw conclusions from assessment data. (A) is incorrect because Stephan is not overrelying on syntactic cues. He is overrelying on graphophonic cues to decode the words in the text. (B) is incorrect. The sentence, as Stephan read it, does not make sense. Therefore, Stephan is not attending to semantic, or meaning, cues. (D) is also incorrect. Stephan is not overrelying on structural cues as he attempts to read the text. (C) is the best answer. Graphophonic cues refer to using knowledge of individual sounds and the letters that represent them to decode words. For example, Stephan reads “sleep” for “sled” and “frog” for “friends.” He is relying heavily on initial graphophonic cues. 63. This question tests your knowledge of phonics. (A) is incorrect because the activity does not conduct a morphological analysis. Morphological analysis is actually a scientific term, not related to reading. (B) is also incorrect. Semantic relationships deal with word meaning, which is not the purpose of this activity. (C) is incorrect. Syntactic structure refers to the structure of a sentence. The activity described best represents the relation of a grapheme and phoneme, making (D) the best answer. Grapheme-phoneme correspondence is the relationship between a written or printed representation of a phoneme, the smallest unit of speech that distinguishes one word from another, and the phoneme it represents, as D and d representing the /d/ in the names.

65

This ebook was issued to Chelissa Gaines, order #10789211026. Unlawful distribution of this ebook is prohibited.

CHAPTER 8

64. This question asks you to show your knowledge of effective comprehension instruction. (A) is incorrect. Looking up words does not guarantee understanding of a concept. (B) is also incorrect. Textbooks have nonfiction conventions, and stories use grammar appropriate to fictional texts. (D) is incorrect. Having students working in pairs when notetaking will only ensure learning for their particular section, potentially missing key information. When instructing students using textbooks, it is critical to introduce concepts and vocabulary so student understanding of the material can increase. Out of all four answers, (C) is the best choice. Having students work together to create a semantic map can help them organize and enhance their understanding of the key concepts.

66. This question asks you to demonstrate knowledge of comprehension instruction, specifically recognizing how to elicit higherorder thinking skills. Although answer choices (B), (C), and (D) are effective strategies, they do not correlate with the task asked by the teacher in this instance. Higher-order thinking skills include connecting previous experiences and knowledge to new information by drawing conclusions, determining relationships between objects or events, and conceptualizing implied ideas. Since the teacher is asking about the feelings of characters, students must infer, which is a higher-order thinking skill. Therefore (A) is the correct answer.

65. This question is evaluating your knowledge of instructional reading strategies. Reciprocal teaching is an instructional activity where the teacher and students conduct a conversation about the selected text. Although answer choices (B), (C), and (D) deal with comprehension strategies and questioning, they do not relate to the process of reciprocal teaching. (A) is the best answer because it starts off with the teacher modeling the types of questions that will best apply to the text being studied. This is the first step in reciprocal teaching.

66

Study Guide for the Teaching Reading Test

This ebook was issued to Chelissa Gaines, order #10789211026. Unlawful distribution of this ebook is prohibited.

Chapter 9

Sample Responses and How They Were Scored

























This ebook was issued to Chelissa Gaines, order #10789211026. Unlawful distribution of this ebook is prohibited.

CHAPTER 9

This chapter presents actual sample responses to the constructed-response questions in the practice test in chapter 7 and explanations for the scores they received. As discussed in chapter 5, each question on the Teaching Reading test is scored on a scale from 1 to 3. The general scoring guides used to score these questions are similar to each other and are reprinted here for your convenience.

General Scoring Guide Score 3 The response demonstrates a thorough understanding of the elements of reading and reading instruction relevant to the question. A response in this category •

Clearly and specifically answers all parts of the question, addressing all crucial aspects of the instructional situation described.



Shows strong knowledge of concepts, theories, facts, procedures or methodologies relevant to the question. Any errors of fact or terminology are minor and do not detract from the understanding shown.



Provides a strong explanation that is well supported by relevant evidence.

Score 2 The response demonstrates a basic or general understanding of the elements of reading and reading instruction relevant to the question. A response in this category •

Adequately answers most or all parts of the question in a way that is appropriate for the instructional situation described.



Shows basic or general knowledge of concepts, theories, facts, procedures or methodologies relevant to the question. Any errors do not detract from the general understanding shown.



Provides a basic explanation that is adequately supported by relevant evidence.

Score 1 The response demonstrates a weak or limited understanding of the elements of reading and reading instruction relevant to the question. A response in this category •

68

Answers some parts of the question at a basic level

Study Guide for the Teaching Reading Test

This ebook was issued to Chelissa Gaines, order #10789211026. Unlawful distribution of this ebook is prohibited.

CHAPTER 9



Demonstrates one of more of the following weaknesses − Fails to answer most parts of the question and/or fails to address crucial aspects of the instructional situation described. − Shows weak or limited knowledge of concepts, theories, facts, procedures or methodologies relevant to the question. The weakness may be indicated by errors or misconceptions. − Any explanation provided is weak and inadequately supported by evidence.

Score 0 The response demonstrates minimal (or no) understanding of the elements of reading and reading instruction relevant to the question. A response in this category •

Fails to respond appropriately to any part of the question.



Shows no knowledge of concepts, theories, facts, procedures or methodologies relevant to the question; or, any information presented about reading and reading instruction is seriously mistaken.

Also receiving a score of 0 would be responses that are blank, completely off topic, or not written in English.

Question 1: Bailey We will now look at three scored responses to Question 2 and see comments from the scoring leader about why each response received the score it did.

Sample Responses Sample 1, Score of 3

One specific reading difficulty Bailey is having is applying comprehension strategies. A recent unit test shows that Bailey was unable to make inferences from the text, nor was she able to make accurate predictions. These weaknesses cause Bailey to frequently call upon the teacher’s help to complete her assignments. An instructional strategy that Bailey’s teacher can use is a think aloud strategy. For example, the teacher can use a text or part of a text to model how to make viable predictions by using clues in the text. After teacher modeling and guided practice, Bailey will begin to independently monitor her predictions for viability. She will also begin to base her predictions on evidence in the text. As a result, Bailey’s predictions will become more and more accurate, thus improving her comprehension. The think aloud and guided

Study Guide for the Teaching Reading Test

69

This ebook was issued to Chelissa Gaines, order #10789211026. Unlawful distribution of this ebook is prohibited.

CHAPTER 9

practice with a graphic organizer strategy can also be used to address inference making. A useful instructional material to help support Bailey’s comprehension development is a graphic organizer. Using a two-column graphic organizer, the teacher and student can record the textual evidence that led to each prediction. This process will ensure that Bailey is basing her predictions on information from the text instead of guessing. Commentary on Sample 1: This sample received a score of 3 because every part of the Task was addressed and answered thoroughly. The test taker identified a specific reading difficulty, comprehension, and referenced evidence in the text to back up how that conclusion was drawn. This sample also shows a deep knowledge of instruction. The test taker identifies an instructional strategy that could be used to target the chosen reading difficulty. The strategy is thoroughly described and the test taker completely addresses how the strategy would improve Bailey’s comprehension. Lastly, the test taker described a useful instructional material that supports the identified reading difficulty. The test taker describes an instructional material and explicitly states how the material will help improve Bailey’s comprehension.

Sample 2, Score of 2

Bailey has a low fluency rate. She can decode words correctly on a sight word assessment, but only if the test is untimed. Her below-average reading rate may be affecting her comprehension. Bailey’s teacher can use paired readings and reader’s theater to increase her fluency. Bailey should also practice reading sight words and phrases at a fast pace until she can recognize them quickly. These activities will improve her fluency. The teacher should use reader’s theater scripts and interesting books at Bailey’s independent level. These materials will give her an opportunity to become familiar with a piece of text that interests her and that is written at a level that is easier for her to decode. Commentary on Sample 2: This sample received a score of 2. All parts of Task 1 and 2 were addressed thoroughly. In Task 1 the test taker provided a basic explanation of Bailey’s reading difficulty using evidence from the information given. In Task 2, an instructional strategy appropriate for addressing Bailey’s reading difficulty is described fully. However, in Task 3 the response does not address how the chosen instructional material will help to improve Bailey’s fluency.

70

Study Guide for the Teaching Reading Test

This ebook was issued to Chelissa Gaines, order #10789211026. Unlawful distribution of this ebook is prohibited.

CHAPTER 9

Sample 3, Score of 1

Bailey’s reading difficulty is that she is unsure of her reading abilities because she has trouble recognizing her sight words quickly. She also does not like to read out loud in front of her classmates. The teacher can help Bailey by letting her practice reading a paragraph before having to read it out loud in front of the class. She will have more confidence that way and learn the words if she practices reading the paragraph a few times. The teacher can use books that have words that are more at Bailey’s level. Commentary on Sample 3: This sample received a score of 1. The responses to all three tasks show a limited understanding of reading instruction. In Task 1 the test taker identifies a reading difficulty, but does not describe it or include any evidence to support the identification. The test taker addresses all parts of Task 2 but in a basic way. Task 3 refers to the teacher using books that “are more at Bailey’s level”, but does not go on to describe how the selection of the instructional material would help to support Bailey’s reading.

Question 2: Diana We will now look at three scored responses to Question 1 and see comments from the scoring leader about why each response received the score it did.

Question 1—Sample Responses Sample 1, Score of 3

Diana’s writing sample gives evidence that she is still at the phonemic stage in the development of spelling knowledge. In this stage, children are making connections between letters and the sounds they represent. Evidence of Diana performing at this stage of writing development can be seen, for example, when she writes “kittin” for “kitten” and “scrach” for “scratch.” Based on the information in this writing sample, the frequency with which Diana is writing in this way and not applying the correct spelling conventions of written language is hindering her ability to communicate with readers. To further develop her writing ability, Diana needs to learn that many words in English are not phonetically regular. In order to help Diana learn conventional spelling patterns, the teacher can provide direct instruction in specific spelling rules. For example, the teacher can teach the CVCe rule by writing examples on the board for Diana to read. Following this explicit instruction, the teacher Study Guide for the Teaching Reading Test

71

This ebook was issued to Chelissa Gaines, order #10789211026. Unlawful distribution of this ebook is prohibited.

CHAPTER 9

can provide Diana with small magnetic letters. The teacher can dictate words in the CVCe pattern and Diana can form them with the letters. The objective of this activity is to give Diana specific reinforcement on this skill. In order to help her retain this information, each word list with a specific spelling pattern can be put into a personalized notebook for Diana to refer to when she is writing. Based on errors made in the writing sample, this strategy should help Diana in correctly writing words such as “take” and “cute” in the future. An informal method of assessing Diana’s progress in writing standard English is by dictating a list of vocabulary containing the particular spelling patterns taught and keeping notes on her performance. Based on the results of these assessments, Diana’s teacher can make decisions regarding what skills need additional direct instruction or reinforcement and which spelling patterns Diana has internalized and is now applying in her writing assignments. Commentary on Sample 1: This sample received a score of 3 because every part of each task was addressed and answered thoroughly. The test taker identified an area that affects Diana’s ability to communicate effectively in writing and referenced evidence in the writing sample to back up how that conclusion was drawn. This sample also shows a deep knowledge of instruction. The test taker identifies an instructional strategy that could be used to target the chosen writing difficulty. The strategy is thoroughly described and the test taker completely addresses how the strategy would improve Diana’s writing. Lastly, the test taker described a way to assess Diana’s progress through the year and how the assessment will be utilized. The test taker provides information regarding how the data will be collected and how it will impact Diana’s future instruction.

Sample 2, Score of 2

Diana’s writing was very hard to read because she is still using invented spelling when she writes. She doesn’t know the correct spelling of many words. For example, in her writing, Diana wrote “bools” for “bowls,” “ruff” for “rough,” and “frnchr” for “furniture.” Diana’s teacher could teach Diana specific spelling patterns in English to help her learn how to spell correctly. The teacher can assess Diana’s spelling by keeping track of her spelling grades on weekly tests. Her grades should improve each week.

72

Study Guide for the Teaching Reading Test

This ebook was issued to Chelissa Gaines, order #10789211026. Unlawful distribution of this ebook is prohibited.

CHAPTER 9

Commentary on Sample 2: This sample received a score of 2. All parts of Task 1 were answered thoroughly and completely. However, in Task 2, the test taker gave one instructional strategy to improve Diana’s ability to communicate effectively, but it was not thoroughly explained and there were no specific references to the writing sample. In Task 3, the test taker gave one assessment tool but did not explain how the data from the spelling tests would be used to impact Diana’s future instruction. Task 1 was thoroughly addressed. However, Tasks 2 and 3 were scored as showing a basic knowledge of the question.

Sample 3, Score of 1

Diana made many spelling mistakes. I had trouble reading this. Diana should study more words. She should take more tests. Commentary on Sample 3: This sample received a score of 1 because responses to all three tasks were very limited. In Task 1 a writing difficulty, spelling, was identified but no references to the writing sample were made by the test taker. Task 2 referred to having Diana study more as an instructional technique, but the response also demonstrated a very limited understanding of how this would help Diana improve her ability to communicate in writing. There also were no references to the writing sample. Task 3 was scored as very limited because the reference to taking more tests indicates the test taker has a very weak understanding of assessment and how data can be collected and used in determining Diana’s future instruction.

Study Guide for the Teaching Reading Test

73

This ebook was issued to Chelissa Gaines, order #10789211026. Unlawful distribution of this ebook is prohibited.

Chapter 10

Are You Ready? Last-Minute Tips

























This ebook was issued to Chelissa Gaines, order #10789211026. Unlawful distribution of this ebook is prohibited.

CHAPTER 10

Checklist Complete this checklist to determine whether you’re ready to take the test. ❏ Do you know the testing requirements for your teaching field in the state(s) where you plan to teach? ❏ Have you followed all of the test registration procedures? ❏ Do you know the topics that will be covered in each test you plan to take? ❏ Have you reviewed any textbooks, class notes, and course readings that relate to the topics covered? ❏ Do you know how long the test will take and the number of questions it contains? Have you considered how you will pace your work?

❏ Are you familiar with the test directions and the types of questions for the test? ❏ Are you familiar with the recommended test-taking strategies and tips? ❏ Have you practiced by working through the practice test questions at a pace similar to that of an actual test?

❏ If you are repeating a Praxis test, have you analyzed your previous score report to determine areas where additional study and test preparation could be useful?

Study Guide for the Teaching Reading Test

75

This ebook was issued to Chelissa Gaines, order #10789211026. Unlawful distribution of this ebook is prohibited.

CHAPTER 10

The day of the test You should have ended your review a day or two before the actual test date. And many clichés you may have heard about the day of the test are true. You should ■■

Be well rested;

■■

Take photo identification with you;

■■

Take a supply of well-sharpened #2 pencils (at least three) if you are taking a multiple-choice test;

■■

Take blue or black ink pens if you are taking a constructed-response test;

■■

Eat before you take the test to keep your energy level up;

■■

Be prepared to stand in line to check in or to wait while other test takers are being checked in.

You can’t control the testing situation, but you can control yourself. Stay calm. The supervisors are well trained and make every effort to provide uniform testing conditions, but don’t let it bother you if the test doesn’t start exactly on time. You will have the necessary amount of time once it does start. You can think of preparing for this test as training for an athletic event. Once you’ve trained, and prepared, and rested, give it everything you’ve got. Good luck.

76

Study Guide for the Teaching Reading Test

This ebook was issued to Chelissa Gaines, order #10789211026. Unlawful distribution of this ebook is prohibited.

Appendix A Study Plan Sheet

























This ebook was issued to Chelissa Gaines, order #10789211026. Unlawful distribution of this ebook is prohibited.

APPENDIX A

Study Plan Sheet See Chapter 1 for suggestions on using this Study Plan Sheet.

Study Plan Content covered on test

78

How well do I know the content?

What material do I have for studying this content?

What material do I need for studying this content?

Where could I Dates planned Dates find the materials for study completed I need? of content

Study Guide for the Teaching Reading Test

This ebook was issued to Chelissa Gaines, order #10789211026. Unlawful distribution of this ebook is prohibited.

Appendix B For More Information

























This ebook was issued to Chelissa Gaines, order #10789211026. Unlawful distribution of this ebook is prohibited.

APPENDIX B

For More Information Educational Testing Service offers additional information to assist you in preparing for The Praxis Series™ Assessments. Tests at a Glance booklets and the Information Bulletin are both available without charge. You can also obtain more information from our website: www.ets.org/praxis.

General Inquires Phone: 800-772-9476 or 609-771-7395 (Monday–Friday, 8:00 A.M. to 7:45 P.M., Eastern Standard Time) Fax: 973-735-0384 or 609-530-0581 E-mail: [email protected]

Mailing Address ETS—The Praxis Series P.O. Box 6051 Princeton, NJ 08541-6051

Overnight Delivery Address ETS—The Praxis Series Distribution and Receiving Center 225 Phillips Blvd. Ewing, NJ 08628-7435

Extended Time If you have a learning disability or if English is not your primary language, you can apply to be given more time to take your test. The Information Bulletin tells you how you can qualify for extended time.

Disability Services Phone: 866-387-8602 or 609-771-7780 (Monday–Friday, 8:00 A.M. to 5:00 P.M., Eastern Standard Time) Fax: 609-771-7165 TTY (for deaf or hard-of-hearing callers): 609-771-7714 E-mail: [email protected]

ETS Disability Services Mailing Address ETS Disability Services P.O. Box 6054 Princeton, NJ 08541-6054

80

Study Guide for the Teaching Reading Test

This ebook was issued to Chelissa Gaines, order #10789211026. Unlawful distribution of this ebook is prohibited.

Study Guide

$22.95 USA ets.org/praxis Copyright © 2011 by Educational Testing Service. All rights reserved. ETS, the ETS logo, LISTENING. LEARNING. LEADING., PPST and PRAXIS III are registered trademarks of Educational Testing Service (ETS) in the United States and other countries. PRAXIS and THE PRAXIS SERIES are trademarks of ETS. 10212

88371-86004 • UNLWEB611

762375